RT dose/volume and Evidence Every Site Flashcards

1
Q

Indications/Dose for gastric and gastroesophageal adenocarcinoma adjuvant EBRT chemoradiation:

Rationale and key studies.
Bonus if you can guess the chemo

A

Concurrent with cisplatin and capecitabine.

Indications
1) Stage:
T3 N+ OR
T3 N0 with positive margins OR
T4 OR N+

ECOG 0-2
2) Adequate nutritional intake
3) Suitable for combined therapy

TOPGEAR (ongoing) looking at NeoAdj 45/25
INT OS/PFS/DM benefit to adj CTRT

BUT CRITICS Trial Peri-op Epirubicin Etop Carboplatin (ECC) c4 then surg then c4 equivalent to ECC surg chemoRT

45/25 (INT Trial) + consider 5.4Gy/3# boost to GTV if can be visualised.
Notify surgeon if both ends of anastomosis likely to be in field.
Chemo

MAGIC (Epirubicin-Cisplatin-5FU), updated to FLOT-4 (5FU-leucovorin-oxaliplatin-docetaxel) (From Neoagis esophagus fame).

How well did you know this?
1
Not at all
2
3
4
5
Perfectly
2
Q

Thyroid RAI doses

A

Iodine 123 study = 2mCi
Ablative dose = 30mCi

ESMO guidelines (US tend to be less):

Adjuvant = 100mCi
micro residual = 150mCi
Metastatic = 200mCi

Retreatment can be considered up to cumulative 600mCi

How well did you know this?
1
Not at all
2
3
4
5
Perfectly
3
Q

Dose EBRT for thyroid cancer and indications:

A

60-70Gy 1.8-2Gy/#

It is optimally used in a small subset of pts w/aggressive locoregional disease.

A single randomized prospective trial failed to recruit adequate patients and only 26 received EBRT. However, a mounting retrospective data showing significant benefit for EBRT in select patients: with gross residual or unresectable locoregional disease, except for patients <45 years old with limited gross disease that is RAI-avid.

How well did you know this?
1
Not at all
2
3
4
5
Perfectly
4
Q

High risk prostate cancer: Definitive Brachy + EBRT dose:

Urinary function requirements?

A

Definitive radiotherapy:
HDR brachytherapy boost, 15Gy/1#
Or 18/2. There is no clear evidence on which dose/fraction schedule is superior, except RCT data suggests multifraction has better local control.

and

46Gy/23# EBRT

and ADT 18-36months (typically 2 years)

HDR brachytherapy can be given before, concurrent with or after the external beam radiation therapy.
The optimal dose, fractionation and brachytherapy scheduling has yet to be defined.

(15 and 15 is easier to remember and is recommended by EVIQ, in US they are way less picky/more aggressive).
IPSS < 15. If IPSS > 15, then formal flow studies
Caution:
Peak urinary flow r< 15 mL/sec or post void residual > 100 mL

How well did you know this?
1
Not at all
2
3
4
5
Perfectly
5
Q

Dose for SABR to spine met (e.g. prostate).

Some indications:
Which tumours are excluded?

A

24Gy/2 fractions (there are a very wide range including 27/3). Range: from 24/1 to 30/5.

Some indications:
Oligigomet disease as defined by COMET-SABR = 4-10
ESTRO ASTRO = 1-5

Oligoprogressive disease

Spinal metastases from the solid primary tumour (excluding haem seminoma and SCLC).
Life expectancy >3months
SINS score<12, >7 get consult.

How well did you know this?
1
Not at all
2
3
4
5
Perfectly
6
Q

For Int Risk Prostate Cancer: Conventional dose vs Hypo#

Evidence for Hypo# dose?

A

78Gy/39#
vs
60Gy/20#

60gy/20# supported by multiple Phase III trials: PROFIT, CHHiP, RTOG - these demonstrate non-inferiority (either with or without ADT, low or intermediate risk) and 2 studies demonstate no increased bladder or bowel toxity, one suggests more grd II/III late toxiciy.

How well did you know this?
1
Not at all
2
3
4
5
Perfectly
7
Q

Extensive stage SCLC dose?

Trial?

A

IF: Any response to systemic therapy and residual thoracic disease.
Limited extra-thoracic metastatic disease burden.

Consolidative dose: 30Gy/10#

CREST Trial:
CTRT improved 2-year overall survival (13% vs 3%) and 6-month progression free survival.

How well did you know this?
1
Not at all
2
3
4
5
Perfectly
8
Q

Limited stage SCLC dose?

Ideally concurrent with?

The alternative approach:

Give a little Hx of this dose:

A

(Concurrent chemotherapy is preferred - ideally with earlier cycles of chemotherapy. Eg cisplayin and Etoposide).

Currently Tiurrisi (INT0096) 45/25 BD remains the standard
66/33 CONVERT Trial found not superior (with trend favouring Turrisis.
Further escalation 70/35 not superior, and potentially more toxic.

Early trials demonstrated an OS benefit w/RT, subsequently confirmed on meta-analysis.
Turrisi 1999 - 45Gy/25# (1.8Gy/#) vs 45Gy BID (1.5Gy/#) (!!!These doses are not BED equivalent!!!) w/concurrent cisplatin etop.

IF Unlikely to tolerate full treatment: 40/15 to 50/25 +/-chemo

How well did you know this?
1
Not at all
2
3
4
5
Perfectly
9
Q

Extensive stage Small cell lung cancer PCI dose:

Discuss evidence.

A

Japanese RCT suggests for extensive stage pts PCI can be omitted in favour of close (3monthly MR) surveillance - original data suggesting benefit defined no cranial disease on CT (not MR) therefore a number of pts who truly had brain mets were included..

BUT Now! An NRG RCT has demonstrated that hippocampus sparing is safe and leads to less neuro cognitive SEs.

Complete responders: 25Gy/5#

Partial responders: 20Gy/5#s

Doses “partly” supported by metaanalysis. Original data (1999) supported OS benefit. RCT data suggests 25/5 as effective as 36/18…

How well did you know this?
1
Not at all
2
3
4
5
Perfectly
10
Q

Extensive stage Small cell lung cancer PCI dose:

A

20/5 or 25/5 both employed, or 25/10 with hippocampus sparing. RCT data = Slotman 2007 - better OS and less symptomatic brain mets at 1 year.

Japanese RCT suggests for extensive stage pts PCI can be omitted in favour of close (3monthly MR) surveillance - original data suggesting benefit defined no cranial disease on CT (not MR) therefore a number of pts who truly had brain mets were included..

Now NRG 25/10 hippocampus sparing minimal neuro cog tox (treated both limited and extensive).

How well did you know this?
1
Not at all
2
3
4
5
Perfectly
11
Q

Dose: Oligo mets to lung?

Criteria?

A

All evidence is at best phase II

48/4 near (<2cm) chest wall.
54/3 if not central. Less radio resistant Mets consider 28/1.
SABR to ultra central appears dangerous (increased haemorrhage). Recent SUNSET trial for NSCLC 60/8, no haemorrhage from 30 pts..

Oligometastatic/oligoprogressive/oligopersistent peripherally located (defined as at least 2 cm from the bifurcation of the lobar bronchi) lung metastases (0-3 metastases).

Tumour/s ≤5 cm in maximum diameter.

Supported by Phase II data:

30/1 or 54/3 - away from chest wall/not central
(phase II study, efficacy of a 30 Gy single fraction was found to have lower 2-year LC compared to multi-fraction SABR (74% vs 91% respectively).

48/4 within 1cm of chest wall - Saffron II TROG (phase II) suggests 48/4 not inferior to 30/1.

How well did you know this?
1
Not at all
2
3
4
5
Perfectly
12
Q

Indication for SABR for NSCLC and dose:

Target volume objectives!!!????!!

Key Study and finding!?

A

The below is from CHISEL which found OS and LC benefit compared with 66/33 or 50/20

1) Stage I-IIa non small cell lung cancer (NSCLC).
2) Tumour ≤5 cm in maximum diameter.
3) Peripherally located tumour defined as at least 2 cm from the bifurcation of the lobar bronchi.6
4) Medically inoperable or declining surgery.

IF tumour GTV <1 cm from chest wall, consider 48 Gy in 4 fractions (12 Gy/fx).

54/3 or 30/1

Target volume objectives!!!
iGTV = DMax 125-143% of PD
PTV= D99% ≥100% of PD.

How well did you know this?
1
Not at all
2
3
4
5
Perfectly
13
Q

Non-small cell lung cancer stereotactic EBRT central tumours?

Define central:

Define Ultra-Central:

Target volume objectives!!!????!!

A

For 1-2 (≤5 cm maximum diameter) N0 M0 non-small cell lung cancer (NSCLC). Centrally located tumour defined as ≤2 cm around the proximal bronchial tree (PBT) - NB not “ultra central”

Ultracentral defined as: PTV touching or overlapping the central bronchial tree, oesophagus, pulmonary vein, or pulmonary artery.

Dose for central 50/5
Dose for Ultracentral 60/8 SUNSET

iGTV should get DMax 125-140% of PD.

PTV should get D95-99% ≥100% of PD.

How well did you know this?
1
Not at all
2
3
4
5
Perfectly
14
Q

NSCLC adj dose in post op patients found to be node positive:

A

Its a bit of a trick: A meta-analysis demonstrates no clear evidence of an adverse or beneficial effect of PORT on survival in patients with pN2 disease. The applicability of this finding to current day practice is questionable. Data from four non-randomised studies suggest a survival benefit for PORT in pN2 disease.

N2 = mets in ipsilateral mediastinal/subcarinal nodes

N2 [R0] disease - 50/25
N2 R1 - 54/27
N2 R2 60/30

How well did you know this?
1
Not at all
2
3
4
5
Perfectly
15
Q

The non-SABR NSCLC curative intent dose:

Evidence?

A

60/30

Bradley (RCT) Trial 2015: for stage III dose escalation to 74Gy not supported. Better survival in 60gy arm

For node negative patients not suitable for SABR consider dose escalation e.g. 66Gy/33# or moderate hypofractionation

How well did you know this?
1
Not at all
2
3
4
5
Perfectly
16
Q

Definitive EBRT dose for FIGO stage IB-­IVA squamous, adenosquamous or adenocarcinoma of the cervix.

A

SIB:
Pelvis and elective nodes: 45/25
Boost nodes 55/25

(CCCMAC)

A dose prescription of 40 Gy in 20 fractions is also in clinical use.

Chemoradiation therapy superior to radiation therapy alone.

How well did you know this?
1
Not at all
2
3
4
5
Perfectly
17
Q

Brachy boost dose for FIGO stage IB-­IVA squamous, adenosquamous or adenocarcinoma of the cervix.

What do you prescribe to?

A

24/3 aim 1-2#s/week depending on timing of brachytherapy in relation to EBRT.

EBRT same as definitive: 45/25,

Prescribe to HR-CTVbrachy (D90)
= 90% of High risk CTV receives at least 100% of the dose.

How well did you know this?
1
Not at all
2
3
4
5
Perfectly
18
Q

Primary curative intent doses for Oral cavity SCC

A

70 to Gross + involved nodes (66/33 if superficial).
63 to High risk = BOT if tongue involved, equivocal nodes.
56 to elective nodes I, II, III, IV
Bilateral i close to midline or within 1cm of tip of tongue or >2 nodes
(Nothing lower than this)

How well did you know this?
1
Not at all
2
3
4
5
Perfectly
19
Q

T1 glottic/larynx

T2?

A

60/25

70/35

How well did you know this?
1
Not at all
2
3
4
5
Perfectly
20
Q

Doses and systemic Tx (where applicable) and key studies for esophagus RT (all the indications):

What is the alternative?

A

ESOPEC. CROSS vs perioperative FLOT. FLOT4 wins 3yr OS 57% vs CROSS 50%

ESOPEC confirms and improves on NeoAegis = Peri-operative MAGIC/FLOT 3yrs OS 56%.

Neoadj:
Neoadjuvant CROSS
- 41.4Gy/ 23#/5, 1.8Gy/#, 5#/week
CROSS (weekly x5 cycles) (CROSS better than FLOT/MAGIC):
- Carboplatin (AUC=2)
- Paclitaxel (50mg/m2)

Definitive RTOG94-05/ INT-0123
- 50.4Gy/28#/5, 1.8Gy/#, 5#/ week
RTOG94-05/ INT-0123 (week 1,5,8,11)
- Cisplatin (75mg/m2)
- 5FU (1000mg/m2/day D1-4)

Palliative 30-36Gy/ 10-12#/5 (36/12 if you can get away with it).

How well did you know this?
1
Not at all
2
3
4
5
Perfectly
21
Q

Preferred salvage option for locally recurrent esophagus Ca (POST multimodal therapy):

A

Esophagus HDR brachytherapy:
25/5 ti 5mm depth and 20mm above and below.
Supported by multple retrospective and institution reports

How well did you know this?
1
Not at all
2
3
4
5
Perfectly
22
Q

The standard of care for medulloblastoma is?

Key factors that decide treatment?

CTVs
VMAT Technique:

A

GTR (90% recur with GTR alone)
Followed by CSI 4 weeks post (evidence says longer time worse outcome).
Dose = 54Gy/30# 1.8Gy/#, with CSI dose determined by risk group:
Average Risk: CSI = 23.4/13, boost post fossa to 54Gy/30
High Risk: CSI 36/20, bring PF to 54.30.

Followed by x4c of CVP (cyclophos, Vincristin cisPlatinum)

All PTVs 3mm
CTVcranial = entire brain + optic nerves + cribiform plate

Boost = GTVpre-op + surgical cavity+gross residual
CTV54 = GTV+5mm, clipped to boundaries

CTV spine: thecal sac, including neural/intevertebral foramen to 1cm below sac (~S1/2)

VMAT: 3 PTVs Cranial, sup and inf spine.

How well did you know this?
1
Not at all
2
3
4
5
Perfectly
23
Q

Critical in plan review of a CSI peads plan:

3DCRT technique for CSI

A

Homogenous cover of the entire vertebrae to avoid scoliosis

How well did you know this?
1
Not at all
2
3
4
5
Perfectly
24
Q

Role of radiation and dose in:
Solitary plasmacytoma

Multiple Myeloma:

A

Plasmacytoma: Radiation is the mainstay, with surgery indicated only where stability is a concern - SINS score >8, Mirrels>8. Data is retrospective only.
Give 45/25 - ISRT - i.e whole involved vertebrae. Or 40/25 if <5cm.
For Extramedullary consider 45/25 to primary and 40/25 ENI

MM:
30/10

Rarely now (most given Mephalan) TBI prior to ASCT

How well did you know this?
1
Not at all
2
3
4
5
Perfectly
25
Q

Radiation dose and rationale for primary CNS lymphoma.

A

Evidence supports a PFS but not OS advantage to WBRT.
The critical treatment is Autologous Stem Cell transplant (if pt can get to that):
Fit young Pt: High-dose methotrexate, Ritux and alk agent - Progression = salvage 36/20. Partial 23.4/13 (These are the same as the CSI doses for medulloblastoma). complete may omit WBRT.

Elderly/not ASCT candidate: Chemo as above -> Partial/complete = 23.4/13, or watch and wait or maintenance e.g tmz
Progression = Individualised care, including WBRT up to salvage 36/20.

Unfit: Curative intent WBRT40/22

Palliative: E.g. 30/10

How well did you know this?
1
Not at all
2
3
4
5
Perfectly
26
Q

Marginal lymphoma doses:

A

Nodal, splenic, MALT (90%, GI 60%, 85% gastric):

Orbital (12% due to chlamydia psittaci - gross) 24/12 (same dose as??, Deb uses 4Gy/2 55% contol), CTV = whole bony orbit
Late tox: epiphora, cataracts,

Gastric MALT: If failure of triple therapy (Claire’s Amplified esophagus) of t(18,11): 30/10 entire stomach (GIJ to duodenal bulb) Fast 4hrs (commonly overnight).

How well did you know this?
1
Not at all
2
3
4
5
Perfectly
27
Q

Kaposi Sarcoma dose and evidence and broad Tx options available:

Key warning to patient about outcome after XRT?
Likely Outcome of RT

A

Surveillance can be considered but indolent progression highly likely (with a larger lesion the to be treated).

  • Surgery: Excisional biopsy is an option for single symptomatic lesion. Can provide sustained local control
  • Radiotherapy: Wide variation of dose (20Gy/10- 30Gy/15#) – higher dose has improved and
    more durable response
  • Topical therapy – limited experience with imiquimod
  • Cryotherapy – mainly cosmetic, no documentation of LC rate
  • Intra-lesion therapy – painful, not generally recommended

Like other sarcomas can take 4 months to regress

XRT - 80-90% LCR but significant risk of progression outside of field.

How well did you know this?
1
Not at all
2
3
4
5
Perfectly
28
Q

A 63 year old man presents with extensive symptomatic Kaposi lesion on his right lower leg extending from just
below the knee anteriorly, involving the calf posteriorly as well as both dorsal and plantar surface of his foot.
c. Describe a suitable radiation therapy technique including a dose and fractionation schedule (4m)

A

I will offer this patient palliative radiotherapy to the right lower leg lesion to a total dose of 20Gy in 10 fractions, 2Gy/X, VMAT technique for local control
* An alternative technique is with water-bath technique, this is cumbersome, with less accurate dosimetry calculation, and on the assumption that the entire ‘water-bath’ (including the leg) is of homogenous
density
Sim
* Position: supine, arms on chest, leg towards gantry, left hip/ knee flex to move left leg out of treatment field
* Immobilisation: cradle under right leg
* Clinical mark-up the symptomatic lesion, or wire the most superior/ inferior/ lateral edge of lesion
* Bolus: 5mm bolus inside cradle, and 1cm bolus on skin anterior to the cradle
* Planning CT: 3mm slice from mid-thigh all the way down to cover the entire foot
Target volume
* CTV20 will be 1cm thickness from skin surface circumferentially from superior to inferior marker, clipped at
bone/ muscle fascia, and spare at least one strip of soft tissue
* PTV20 = CTV20 + 5mm expansion
Technique: partial arc VMAT with 6MV photon
Plan review:
* Ensure PTV well-covered by 95% isodose line
* Minimise hotspot outside of PTV
* Minimal OAR at risk of concern; important to ensure contralateral leg is not receiving any dose
Treatment verification: daily kV imaging, with 5mm tolerance

How well did you know this?
1
Not at all
2
3
4
5
Perfectly
29
Q

Dose, and rationale for adj XRT post node dissection for melanoma. Volumes:
For whom should you consider it?

A

Burmeister study: Improve LC from 60-80%
CTVprimary - 2cm margin around scar
CTVnodal - 3cm prox-distal (like esophagus) and 0.5cm axial.

EVIQ CTVs: Pre-operative disease + surgical bed (including scar) ± margin as appropriate for potential sub-clinical disease. Consider non-surgically perturbed at risk nodal basin

Dose 48/20
UNLESS H&N, Groin lymphoedema - 50/25

Burm’s criteria at least 1 RF:
1 or more parotids, 2 or more neck (pr>3cm), 3 or more groin (or >4cm), ENE

How well did you know this?
1
Not at all
2
3
4
5
Perfectly
30
Q

Dose technique for average risk meduloblastoma

A

Avg = age>3, GTR w<1.5cm, not mets

I will offer this patient cranio-spinal irradiation to a dose of 23.4Gy in 13 fraction followed by tumour bed boost to total dose of 54Gy/30#) (1.8Gy/#, 5#/ week). VMAT technique with 6MV photon- 3 isocentres (cranial, upper spinal, lower spinal), junction at 4cm depth, no feathering.

Pre-SIM
* Ensure recovery from surgery (but within 4 weeks!!)
SIM
* Positioning: supine, arms on side, head towards gantry
* Immobilisation: on vac bag/ body cradle, thermoplastic mask
* CT simulation: 3mm slice planning CT from vertex to mid-thigh
* Fusion: with pre-op MRI, post-op MRI
Target volume:
* Phase 1: CSI 23.4Gy/13# (entire intracranial/ spinal subarachnoid volume)
o CTVcranial = Entire brain, covering cribriform plate, and optic nerve
o CTVspine = Thecal sac expand laterally to intervertebral foramina; inferior level must be determined on
MRI (usually ~ S1-2)
o PTVcranial = CTVcranial + 3mm
o PTVspine = CTVspine + 5mm lat + 10mm AP + 10mm SI
* Phase 2: Tumour bed boost 30.6Gy/17#
o GTV: surgical cavity + post-op residual disease
o CTV30.6: GTV + 5mm
o PTV30.6: CTV30.6 + 3mm

How well did you know this?
1
Not at all
2
3
4
5
Perfectly
31
Q

Describe a suitable radiation therapy technique and dose fractionation schedule for 3cm Merkel cell carcinoma completely excised (R0) from the upper right lateral arm
(over the deltoid)

Give benefit, T1 vs. T2

A

SEER database suggests likely OS benefit to PORT. T2 = 2-5cm (consider PET even in T1 - 25% T1 and T2 get upstaged).

Definitive EBRT alone 54Gy in 27 fractions, 2Gy per fractions to the right deltoid primary and axillary node + supraclavicular fossa to improve local and distant control, and overall survival.
Pre-SIM:
- MDM discussion
Simulation:
- Position: supine, arms akimbo, head towards gantry
- Immobilisation: vac bag, knee fix, ankle support
- Addition: wire scar, bolus 3-5 cm around scar/ primary lesion
- Planning CT: 3mm slice planning CT from upper cervical to below diaphragm (cover the entire lung for DVH)
Fusion: pre-op MRI/ PET
Target volume:
- CTV54 = (pre-op GTV and scar) + 4cm radial margin, and 1.5cm DEEP to skin down to fascia, clipped at
anatomical boundaries
- CTV51.2 (50Gy EQD2) = CTV54 + at risk nodal group (level 1-3 axilla and supraclavicular fossa)
- PTV = CTV + 1cm margin for setup uncertainties
Technique: 5-7 field IMRT technique with 6MV photons
Plan review
- Target coverage, PTV D98>95%
- Minimise hotspot PTV D2<107%, ensure no hotspot outside of PTV or in OAR
- Ensure low dose (10%, 50%) reasonably distributed around PTV
- Try to spare a strip of skin/ subcutaneous tissue
- Review OAR DVH
o Brachial plexus Dmax< 54Gy
o Ipsilateral lung: V20Gy<30%, V30Gy<20%
Treatment verification: daily CBCT, matched to bone, soft tissue review

How well did you know this?
1
Not at all
2
3
4
5
Perfectly
32
Q

Describe a suitable radiation therapy technique and dose fractionation schedule for TBI. Justify your answer

A

DOSE PRESCRIPTION
* 12Gy in 6 fractions, 2 Gy per fraction, 2 fractions per day, over 3 days
* Prescribed to a single point at midline of the patient (usually umbilicus)
* (Other dose options: 2Gy/1#, 13.2Gy/11#, 4Gy/2#)

Pre-SIM
* MDM discussion
* Fertility preservation referral

SIMULATION
* Position: supine, upper arms on side resting on 4cm polystyrene (maximise lung shielding from lateral beam),
and hand resting on abdomen
* Planning CT: 3mm slice covering the entire body length (from vertex to mid-thigh)
TARGET VOLUME
* Entire body contour
TECHNIQUE
* 4-field equally weighted MV photons AP/PA and opposing lateral, with extended SSD (4m) and largest practical
field size
* 6/10/18 MV (avoid 18MV if possible)

Alternate APPA and oppose lateral for each fraction (e.g. APPA for fraction 1, 3, 5, lateral for fraction 2, 4, 6)

Lateral field Position:
* patient lie supine, small sponge under head, knee fix,
* Upper arm resting on 4cm polystyrene (to reduce lung dose)
* Hand resting on abdomen
* Trolley turn around for the opposing lateral field treatment
Compensator/ bolus (‘beam spoiler’) – (because of skin-sparing effect of photon)
* Super-flab – on lateral and anterior surface of neck + chest (to reduce dose to lung)
* Perspex – as head frame compensator, and from mid-thigh inferiorly (thicker from mid knee
inferiorly)

How well did you know this?
1
Not at all
2
3
4
5
Perfectly
33
Q

Doses and presim for pituitary adenomas:

A

SRS (non-functional) 14-16 Gy/ 1#
SRS (functional) 20 Gy/1# Functional needs higher dose

Fractionated SRT 25 Gy/ 5# (5 Gy/#)
(BED~ 11-13Gy / 1#)
(FSRT if: lesion >3cm size, or <3mm from optic

IMRT/ VMAT 40Gy/ 15# (1.8Gy/#)
50.4 Gy/ 28# (1.8Gy/#)

PRE-SIM
* Baseline ophthalmology, neurology, and endocrinology review

How well did you know this?
1
Not at all
2
3
4
5
Perfectly
34
Q

Bone SABR:
Rationale
Contraindications
Dose
Technique

A

24Gy/2#
Rationale - Compared to multi# EBRT, multiple RCTs demonstrate higher complete response rates at 3 months, BUT higher vert body collapse 17% vs 10%.
In oligomet setting - COMET SABR shows OS benefit,

Contras: Previous radiation at site!! (No evidence to support this), SIN Score > 11. >3 spinal mets (i.e. much higher risk of collapse)

Technique:
sim CT + MRI fusion
8-Field co-planar, non-opposed 6Mv photons
Prescribe to covering isodose

Volumes:
GTV on fusion
CTV = GTV + CTV defined by International Spin Radiation Consortium (ISRC) guidelines.
E.g Well lateralised Vert body then CTV = whole vert body + ipsi pedicle. e.g 2. Diffuse vert body, CTV whole v.body bilateral peds.
PTV = CTV +2mm, EXCEPT trimmed off spinal cord!!!
Verification daily 3D CBCT to bone.

How well did you know this?
1
Not at all
2
3
4
5
Perfectly
35
Q

Dose, volumes and rationale for RT in soft tissue sarcoma:

Key studies:

A

Key studies: O’sullivan = RCT of pre vs PORT
STRASS = retroperitoneal dont do RT unless relapse (if you have to to 50.4/28)
SEER database
Rationale: For higher-grade STS SEER multistudy reterospective analysis supports a benefit to RT in conjunction with surgical resection. Both pre and post have equivalent OS but differ in their toxicities - E.g higher wound complications (reversible) in pre vs higher fibrosis rates in PORT (not reversible). RCT supports Brachytherapy as approx equivalent LCR.

For reteroperitoneal tumours surgery alone (STRASS). PORT not recommended, consider if recurrence.

For extremities:
Pre-Op: 50.4/28 (Myxoid liposarcoma 36/12)
Some data demonstrating equivalent outcomes with 30/5!!
GTV as delineated on all imaging (MRI T1+C)
CTV = GTV + 1.5cm radial, 4cm (or 3cm) sup-inf
PTV= CTV +1cm

PORT:
CTV60= pre-op GTV/tumour bed +1.5cm radial (as above) + !2cm! sup inf
CTV50= CTV60+2cm sup-inf
PTV = CTV50+1cm

Definitive:
At least 70Gy required for local control

How well did you know this?
1
Not at all
2
3
4
5
Perfectly
36
Q

Dose and technique for Hodgkins (and rationale/evidence and clinical information used to decide dose).

Key Chemos?

A

(except for popcorn cell nodular lymph prodominant HL)
Relevant studies in order of disease severity: HD10/EORTC, HD11, RAPID, HD15 (Stag III/IV).
Risk = AMEEN = Age>50, Mediastinum>1/3rd, ESR elevated, Extra Nodal, Nodes>3. (1 or more = HR stg I/II).

ISRT: GTVp = residual, GTVpre = prechemo. CTV=GTVp+5mm axial, and cranio-caudal expansion to cover GTVpre. If no GTV pre then 1.5cm.

Stg I/II and low risk (H10):
1) x2ABDV then ISRT 20Gy (10yr PFS 10%) is standard
2) PET direct RAPID (fav or Unfav) x3ABVD no need for RT if PET-ve post c3.
IFF +ve then x1ABVD the 30Gy ISRT

Stg I/II and HR (AMEEN>0) H11 (10yr PFS 85% same as fav):
1) x4ABVD the 30Gy ISRT (i.e 10Gy more)
2) PET direct RAPID (fav or Unfav) x3ABVD no need for RT if PET-ve post c3.
IFF +ve then x1ABVD the 30Gy ISRT

Stg III/IV (HD15): BEACOPPx6 (dose escalation to x8 kills more ppl than it helps). IFF PET +ve or residual >2.5cm then 30Gy.
Exception: urgent debulking of initially bulky - 30/15.

ABVD = Adriamycin, Bleomycin, vinBLASTine, Dacarbazine - 1 month cycles, 2 infusions/mth

BEACOPP= Bleo, etoposide, Cyclophosphamise, Oncoven (vincristine), procarbazine, pred.

How well did you know this?
1
Not at all
2
3
4
5
Perfectly
37
Q

What are the subtypes of Hodgkins, give dose/approach for the exception to the group:

A

HL = RS cells (CD 15 and 30)
or Popcorn cells (CD 19,20 like NHL, and neg CD 15/30)

Classical has 4 types (2 are assoc w/EBV):
1) Nodular sclerosing (70%) often mediastinum M=F broad band of bifingent sclerosing collagen
2) Mixed Cellularity (20%) EBV associated, often abdo/Spleen, M>F, Diffuse effacement of node with mix of lympocytes/plasma/eosin and RS Cells. No sclerosis
3) Lymphocyte Rich (5%): best prognosis
4) Lymphocyte Depleted EBV associated.

Nodular Lymphocyte Predominant HL (5%) CD 19 and 20 (Ritux target). Frequently transform into DLBCL
Stage I/II get upfront ISRT 30Gy then R-CHOP if PET +ve
III/IV get R-CHOP +/I ISRT

How well did you know this?
1
Not at all
2
3
4
5
Perfectly
38
Q

Dose/schedule and rationale for bladder preservation approach.

Evidence for chemo?

A

Bladder preservation approach offers chance of preservation (66% at 8 years) with comparable OS to neoAdj chemo + surgery (both in order of 75%). Combined analysis of 2 trials favours hypofractionated approach for better LCR ~60%.
Concurrent chemo is supported by multiple phase II trials that demonstrate LC benefit, one trial had a trend for OS benefit

EBRT, Vmat technique 55Gy/20#, 5#s/week, to bladder and tumour volume (no clear evidence for ENI), 6MV photons, prescribed to D50. Concurrent with weekly cisplatin 35mg/m.sqr.

How well did you know this?
1
Not at all
2
3
4
5
Perfectly
39
Q

Dose, technique and volumes for bladder preservation approach.

A

EBRT, Vmat technique 55Gy/20#, 5#s/week, to bladder and tumour volume (no clear evidence for ENI), 6MV photons, prescribed to D50. Conc w/Day 1 MMC 12mg/msq, and D1 and D15 5-FU 2.5g/msq (DFS BENEFIT, trend to OS).

Pre sim: EMPTY BLADDER and Rectum. Maximal TURBT.
Sim: supine head, towards gantry, vac bag immobilisation, w/knee/ankle fix.
2mm Slice CT+contrast mid-abdomen to upper 1/3rd femur. Fuse w/pelvic MRI.

GTV: Gross tumour volume on imaging/cystoscopy, or is complete TURBT than pre TURBT volume.
CTV = GTV+5mm on gross extravesicular extension, whole bladder, IF: bladder neck/trigone, CIS, multi tumour do prostate, or females 2cm urethra. IF posterior tumours ant vag wall.

How well did you know this?
1
Not at all
2
3
4
5
Perfectly
40
Q

Broad Mx approach to germ-cell ball cancer

Name some factors in Stg 1 that may influence your choice.

A

(never Bx)
Orchidectomy with high spermatic cord ligation
The Mx based on seminoma vs NSGCT:
NSGCT: Any residual then resect. Lmited role for RT, LVI is critical (relapse 50 vs 15%), LVI = BEPx1, LVI neg = Surveillance.

Seminoma Mx based on Stg (Stg 1 = TanyN0, IIa=N<2cm = N1):
Stg 1 = Surveillance (15% Recurrence, but no OS risk) vs 1-2 cycles of carboplatin (unclear data for 1vs2) vs Para-aortic 20/10.
The TE19 study suggests Chemo is as effective as RT (relapse rate 5%), but less toxic.
Factors that may make surveillance bad = tumour>4cm, involves scrotum, raised markers post surg, poor compliance with follow up

Stg II - IIa (single node<2cm) consider dog-leg RT 20/10 (30/10 to dose).
Otherwise BEPx3 (or EPx4 if cant do B):
BEP = Bleomycin (pulmfibrosis risk) Etoposide

How well did you know this?
1
Not at all
2
3
4
5
Perfectly
41
Q

Mx post chemo residual ball cancer

A

<3cm do surveillance, >3cm do PET at 6 weeks. If neg surveillance, if+ resection (preferred) or RT

How well did you know this?
1
Not at all
2
3
4
5
Perfectly
42
Q

Suitable dose and broad techniques (and rationale) for Ball cancer:
1) Node -ve
2) Single small node (<2cm)

A

Rationale: Decrease relapse rate from 15% to 5%
1) EBRT VMAT technique treating para-ortic nodes to 20G/10#s, 6MV photons, prescribed to D50.
CTV20= IVC and aorta, from 2cm below renal apex to aortic bifurcation, expanded by 2cm and trimmed to anatomical boundaries. PTV = 0.5cm

2) Dogleg: As above but include common, internal, external and iliac vein to UPPER-BORDER of acetabulum

How well did you know this?
1
Not at all
2
3
4
5
Perfectly
43
Q

For pancreatic cancer give the: Benefit/evidence and doses of EBRT for each situation it may be used.

A

Inoperable panc cancer has 5yr OS of 5%, operable has 25%

One key role therefore is in borderline inoperable - where metananlysis suggests neoAdj ChemoRT may make 1/3rd operable. BUT the specific role of RT is unclear (PREOPANC 2), with 36/15+ gemcitabine -> surg ->x4cycle gem = x6FOLFIRINOX (both approaches are highly toxic w/50% Pt having “serious adverse events”).

Operable:
Resecection - if +ve margin metanalysis supports a LC benefit to 45/25.

Inoperable:
Very limited evidence of OS benefit to the addition of RT to Chemo - 50.4/28
MASTERPLAN trial is looking at 4 cycles mFOLFIRINOX SABR with chemo: 40/5

How well did you know this?
1
Not at all
2
3
4
5
Perfectly
44
Q

Target volumes for pre-op extremity soft tissue sarcoma:

A

Fusion: MRI and FDG PET
Target volume:
- GTV = visible disease based on planning CT, and fused MRI/ FDG PET (T1 and T2 sequence + contrast)
- CTV = GTV + 1.5cm radial expansion + 4cm sup-inf expansion (include all oedema on T2 sequence)
- PTV = CTV + 1cm expansion

How well did you know this?
1
Not at all
2
3
4
5
Perfectly
45
Q

Role and doses for Osteosarcoma?

A

Free knowledge: Histological Dx - characterised by osteoid production, and classified by site (central = within = conventional = 90% high grade, Juxta cortical most low grade, Extraskeletal is rare).

RT only if:
1) Cannot be resected - neoAdj Chemo->70Gy->Adj Chemo (3 agents including methotrexate)
2) Further resection not possible:

How well did you know this?
1
Not at all
2
3
4
5
Perfectly
46
Q

Roles and doses of radiation for mesothelioma:

A

Main role
Palliative 20Gy/5# low dose equivalent to HD

Other:
1) Neoadjuvant - investigational

2) Adjuvant:

NOT if Extra pleural pneumonectomy (EPP)! Consider but don’t for pneumonectomy and decortication. RO 50/25
R1 50/25 and boost to 56/28.

Phase 2 (MARS) says no benefit and may cause harm after EPP
May be tolerated, await IMPRINT trial to see if any benefit.
* SABR for oligo-mets

No role
Prophylactic RT to procedural tract

How well did you know this?
1
Not at all
2
3
4
5
Perfectly
47
Q

Partial breast: Who, dose, technique, rationale:

A

At least 7RCTs have shown non-inferiority to WBRT
IMPORT LOW: Unifocal. Age>50, IDC grd1 to 3, <=3cm, <3nodes (N1)

40/15, 5#/week, 3Dplanned tangents with FIF, Prescribed to ICRU83, 100% PTV>95%.
Sim: breath hold - all that shit
Volumes:
Tumour bed on imaging (clips, wired scar)
CTV=TB + 15mm, trimmed 5mm off skin and lung interface.
PTV=CTV+10mm.

How well did you know this?
1
Not at all
2
3
4
5
Perfectly
48
Q

For lung SABR define the doses in terms of tumour location:

A

Peripheral: >2cm from proximal bronchial tree = 54Gy/3#s UNLESS <2cm from chestwall - then 48Gy/4#s

Central: <=2cm around proximal bronchial tree, but not ultra central. 50Gy/5#

Ultracentral: PTV overlaps: proximal bronchial tree, pulmonary artery/vein, or esophagus. Controversial whether to treat with SABR - some reports of fatal pulmonary haemorrhage. SUNSET Trial 2024. Prospective phase 1 - 30Pts good LCR and no haemorrhage with 60Gy/8#s.

How well did you know this?
1
Not at all
2
3
4
5
Perfectly
49
Q

Curative approach (and rationale) to a pancoast tumour (AKA?):

A

Superior sulcus tumour: 2 phase II trials show pre-operative chemoRT has superior DFS and OS compared to historical controls. Approx 50% 5yrOS. But other phase II data suggests chemo alone may be sufficient…
DO ONLY if resectable (

How well did you know this?
1
Not at all
2
3
4
5
Perfectly
50
Q

Give a dose prescription for SABR (doesnt matter which dose, the main bits are prescribed to and aim):

A

Curative intent SABR 50Gy/5#s (using central tumour), 1#/day, prescribed to 100% isodose, 6MV photons, DCAT technique.
Aim Dmax within PTV 125-140% of prescribed dose, CI-100; 1.2-1.3, CI-50<5. PTV D98>100%PD.

CI-100 = Ratio of Volume getting 100% of prescribed dose to volume of PTV. Should be 1.2-1.3
CI-50 = Ratio of Volume getting 50% of prescribed dose to volume of PTV. Should be <5.

How well did you know this?
1
Not at all
2
3
4
5
Perfectly
51
Q

Technique options for SABR

For the moving target which may be better. Describe it in detail.

A

Technique options
- Fixed beam >7 beam – fixed beam angle, converging on one isocentre, with open field, mixed coplanar/ non- coplanar
- DCAT technique – modulated; open field; doesn’t allow MLC covering PTV at any stage; preferred for moving target
- VMAT technique – static target

DCAT (dynamic conformal arc therapy)
- MLC only modulate field edge, and doesn’t cover the PTV at all time (compared to VMAT where MLC can cover PTV at some)
- Avoid Interplay effect i.e. shielding of PTV with MLC during VMAT, leading to underdosage of PTV due to tumour motion
- VMAT is ok on apex

How well did you know this?
1
Not at all
2
3
4
5
Perfectly
52
Q

Primary Tx for a T2NOMO nasopharyngeal carcinoma:

What 2 factors may change volumes?

A

EBRT (VMAT partial arc 6MV photons) alone:
CTV_70Gy/35# = GTV+5mm

CTV_63/35 (EQD2 60Gy) = Entire nasopharynx +
Sup: Skull base w/ovale and rotundum, inf sphenoid sinus
if >=T3 - whole sphenoid and cavernous sinus
Inf: Soft palate
Ant: posterior 1/3 of max and nasal sinus
Post: Bilateral retropharynx nodes, ant 1/3 of clivus (whole clivus if involved - i.e. T3)
Lateral: Pterygoids and parapharyngeal space.

CTV 56 (EQD2 50): CTV 63+ Levels II-V.
If node +ve include 1B.

How well did you know this?
1
Not at all
2
3
4
5
Perfectly
53
Q

Treatment approaches for Primary CNS Lymphoma (with doses).

A

Divide into, young, elderly, and fit and unfit. Then response based (Autologous stem cell or WBRT or more chemo).
Evidence: post chemo WBT improves PFS but unclear OS benefit.
Unfit (for chemo): 40/20 WBRT or less

All fit (young n old) should get HDmethotrex + ritux + Alkylating agent (eg TMZ). +/- further chemo then:
If response: ASCT (preferred in young) or WBRT 23.4/13
No response/prog: Isofosphamide vs 23.4/13 then boost residual to 45/25

Fit elderly - 3 options:
WBRT
Maintenance TMZ
or Wait and Watch

How well did you know this?
1
Not at all
2
3
4
5
Perfectly
54
Q

The OARS of head and neck and Dmax

A

Brain stem/ optic chiasm = Dmax <54Gy

Optic nerve Spinal cord = Dmax <45Gy

Temporal lobe/Brain = Dmax< 60Gy, V40Gy<5cc

Brachial plexus= Dmax< 66Gy

Retina/ eye = Dmax< 50Gy
Lens = Dmax < 8Gy

Lacrima = Dmax < 30Gy

Manidble = Dmax < 70Gy

Carotid = Dmax <100Gy

Parodid is a mean dose = Dmean 26Gy

How well did you know this?
1
Not at all
2
3
4
5
Perfectly
55
Q

Describe suitable follow up regimes for Anal SCC, and some key principles.

A

Most recurrence is locoregional, so the role of CT CAP is unclear.
“Ass”ess at 8-12 weeks
* Clinical review at 6-8 weeks to ensure clinic complete response.
If incomplete continue as normal but at 26weeks residual/progression should be Bx and plan for APR.
* Clinical follow-up (DRE, and endoscopy surveillance) 3 monthly 1
st year, 6 monthly 2nd year, annually until 5
years
* 3-month MRI and PET
* Consider annual CT C/A/P if T3-4 or N+

How well did you know this?
1
Not at all
2
3
4
5
Perfectly
56
Q

Dose and volumes for Anal SSC, what is the chemo?

A

> T1, then chemo= MMC + (5-FU or Capecitabine) (MMC essential, substitute for cisplatin as last resort) for improved LC and CSS

SIB technique:
CTV50or54: primary + 2cm+ whole canal/sphincters

CTVnodes: inguinal, mesorectal, ischiorectal, pre-sacral, internal (inc obturator and external ilacs - do not do common.

Non bulky (<4cm = <=T2): 50.4Gy/28 and
42Gy/28
- COVID Dose 45/15
Bulky (T3): 54/30 and 45/30.

How well did you know this?
1
Not at all
2
3
4
5
Perfectly
57
Q

Describe suitable follow up regimes Post rectal (stg I to III) Ca Mx.

A

1st year:
3monthly: Physical exam/Hx
6monthly:
- CEA for 2 yrs then annually for 2 yrs
- DRE and sigmoidoscopy
CT CAP at 1 year.

2nd Year:
24month - Hx/Ex, DRE/Sigmoidoscopy

3rd-5th years:
Annual Hx/Ex
Coloscopy every 3-5year

How well did you know this?
1
Not at all
2
3
4
5
Perfectly
58
Q

Key anal cancer studies:

Key principles in the timing of anal SCC XRT:

A

Nigro (1974) Retrospective study 30/15 + CTx (MMC+5FU)

ACT1: Chemo improves LC and CSS compared w/RT alone. Other trial says cant omit MMC. No benefit to induction chemo.

ACT2: MMC+5FU = MMC+cis isoeffective/isotoxic

PLATO recruiting=ACT3 (T1 surg +/- adj), ACT4 (<4cm de-escalation 41.4Gy/ 23 = butt hole CROSS), ACT 5 (N+ and/or T>=3, boost to 61Gy).

OTT should be minimised (Not>6wks)
Treatment breaks are shit for tumour control avoided - BD or treat on weekend if any missed #.

How well did you know this?
1
Not at all
2
3
4
5
Perfectly
59
Q

Concurrent chemotherapy for butt SCC?

A

5FU/ MMC (MMC IV D1 12mg/m2 + 5FU infusion 1000mg/m2/day D1-4 week 1 and 5

How well did you know this?
1
Not at all
2
3
4
5
Perfectly
60
Q

Rationale for radiotherapy and chemoradiotherapy in dick cancers:

A

The goal is organ preservation.
Benefit of concurrent chemo is extrapolated from vulva/anal.

For adj Pelvis RT retrospective data is mixed. Some suggests DSS and OS benefit, especially where multiple nodes. Therefore, adjuvant RT is recommended following a PLND with +ve nodes.

How well did you know this?
1
Not at all
2
3
4
5
Perfectly
61
Q

Doses and volumes for penile cancers:

And benefit for each.

Also, indications for concurrent chemo

A

Use 2gy/# cisplatin 40mg.
1) Definitive local:
- Primary <4cm and does not involve cavernosum local +/- cisplatin - 60Gy/30#s CTV = GTV+2cm (alternative is brachy 60/5). Surgery seems about equivalent for local - 5yr OS 75%, 80% LCR.

2) Definitive advanced (surg is the standard approach):
>4cm, or >=T3 (meaning cavernosus invasion or worse): 50/25 whole penis and inguinal/pelvis, boost primary (+ve nodes, or ece and dissection) to 66/33Gy. This supported by UK RCR.
(PORT is the same but adds 60/30 to micro)

> 4cm or >T2 should always get concurrent chemo:
E.g. cisplatin or cisplain + MMC

3)PORT: 66/33 to residual, 60/30 to R1and ECE, then 50/25 to everything else. Chemo is R1, ECE or very advanced. Benefit extrapolated from anal/cervix. RT added to CT may increase PFS to 30% from 15% (multi-institutional anaylsis).

4) NeoAdj is investigational InPACT (45/25).

How well did you know this?
1
Not at all
2
3
4
5
Perfectly
62
Q

Broad statement on penile SCC node status, general Mx of penile Ca and evidence.

Indications for treating nodes with RT:

A

Approximately 20–30% of patients with positive inguinal nodes have positive pelvic nodes. Lymph node status is a major prognostic factor for penile cancer.
Surgery is the mainstay of locoregional treatment.
There is a lack of high-level evidence to guide management.

ECE or >2 nodes

How well did you know this?
1
Not at all
2
3
4
5
Perfectly
63
Q

What does Adj RT add to chemo for dick cancer after inguinal surgery?

A

A multi-institutional retrospective analysis was carried out to review the benefit of adj RT to Adj Ctx after inguinal surgery for penile cancer. Found longer cancer-specific survival with ChemoRT (29%) compared with adj chemoalone (16%)

How well did you know this?
1
Not at all
2
3
4
5
Perfectly
64
Q

Role of chemo in dick cancer:

What is the trigger point for neoAdj and the level of evidence.

A

NeoAdj:
1) Bulky: resectable disease rarely cured with a single modality; therefore, consider NACTx prior to Iliac LND.
2) >2 nodes or bilateral nodes - Phase 2 support OS benefit to TIP (paclitaxel, ifosfamide, and cisplatin) or “TIP PIC”.

Adj:
No diff bet adjuvant versus neoadjuvant chemotherapy or adjuvant versus no intervention, OS was higher with neoadjuvant chemotherapy compared to no intervention and PFS was higher with adjuvant compared to neoadjuvant chemotherapy.

How well did you know this?
1
Not at all
2
3
4
5
Perfectly
65
Q

Types of vaginal brachy applicators (and role of each):

A

Vaginal cylinder (often segmented) - best for symmetric brachy

Multichannel cylinder - For asymmetric dose (also e.g. to avoid bowel loop 80Gy)

Tandem and ovoid or ring applicator (or tandem or ring): Cervix cancer

Combined interstitial intracavity devices.
Rotte applicator (a 2 channel tandem thing where both channels can go into the uterine cavity) - in rare case of intact uterus.

How well did you know this?
1
Not at all
2
3
4
5
Perfectly
66
Q

In general terms, list the steps in treatment planning LDR prostate implant (3 marks)

A

Planning:
- Based on TRUS volume
- Contouring
o CTV = prostate; PTV = CTV + 3mm (except 0mm posterior)
o Urethra and rectum
- Placement of seed ~ 100 through the prostates
- Review DVH
o PTV: V100>98%, V150% ~50%, V200%~ 13-18% , D100%>95Gy
o Urethra: D10<210Gy, no 150% isodose line encircling/ in urethra
o Rectum: V100<1cc, 70% isodose line away from anterior rectal wall
- Seed ordered from overseas

How well did you know this?
1
Not at all
2
3
4
5
Perfectly
67
Q

In general terms, list the steps in treatment that are involved with delivering an LDR prostate implant (3 marks)

A
  • GA, lithotomy position
  • Prep and drape
  • IDC and injection of ~120mL mixed normal saline/ contrast
  • TRUS probe per-rectum, ensure image closely resembles volume study
  • Interstitial needle inserted using template and under ultrasound guidance, and position
    checked
  • Stylet (with radioactive seed) deployed
  • Interstitial needles removed, leaving stylet (with radioactive seed) in situ
  • TRUS probe removed
  • Cystoscopy – to check bladder and urethra of any seed, trauma, obstruction
How well did you know this?
1
Not at all
2
3
4
5
Perfectly
68
Q

For Ewing’s give the indications for PORT and approach (dose/volumes). How does the PORT differ from definitive?

A

PORT dose = definitive (post chemo) dose. Post operative volumes based on degree of necrosis (<90% necrosis = bigger volume = pre chemo GTV).

Indications for PORT:
* Positive (R1/R2) or close margin <1cm
* poor response to chemotherapy (<90% necrosis)
* tumour spill
* For tumour spill, boost pre-chemo volume

45Gy/25# to pre-chemo volume then boost to 54/30 as below (I.e the only thing that changes is bigger volume if worse/less necrosis):
IF post CTx NECROSIS >90%, then boost 9Gy/5# (1.8Gy/#) to post-chemo GTV (smaller volume)

IF NECROSIS <90%, then boost 9Gy/5# (1.8Gy/#) to pre-chemo GTV (bigger volume)

How well did you know this?
1
Not at all
2
3
4
5
Perfectly
69
Q

For Ewing’s XRT (PORT or curative), what are the planning considerations?

A
  • Do not treat across a joint/ encompass an extremity circumferentially (i.e. need to spare a strip), unless absolutely necessary for tumour coverage
  • Reduce margins if there is no extension beyond joint space, but adjacent epiphysis is in volume
  • For diaphyseal lesion, exclude one epiphysis of the affected bone, if possible
  • If CR to chemo, boost post-chemo volume
  • If incomplete response then boost pre-chemo volume
How well did you know this?
1
Not at all
2
3
4
5
Perfectly
70
Q

For Ewings, what is the chemo, and timing of chemo in relation to XRT in the following settings:

Definitive
Adjuvant

A

VDCA-IE – VDCA+IE better than VDCA alone in CCG-7881 trial (5yr OS 72% vs 61%)

Induction chemotherapy 6 cycles Q2W (=12 weeks)
o Chemotherapy break for surgery (week 13)
o Adjuvant chemotherapy (restarted as soon as possible) to 48 weeks (i.e. total of 24 cycles)
o If definitive/ adjuvant RT, start with cycle continue chemotherapy (withhold doxorubicin during RT)

How well did you know this?
1
Not at all
2
3
4
5
Perfectly
71
Q

Not a dose/technique question. But very high yield.
For each tumour marker, state which tumours can elevate it (i.e. not all of these are routine by any means).
1) CEA
2) CA15.3
3) CA 125
4) CA19.9
5) AFP. Give half-life
6)Beta or just hCG. Give half-life
7) LDH
8) Calcitonin
9) NSE
10) S100

A

1) CEA: Colorectal Ca, colangiocarcinoma, medullary thyroid
2) CA15.3: Breast
3) CA 125: Ovarian
4) CA19.9: Pancreatic Ca
5) AFP: NSGCTs = yolk sac and embryonal, Liver HCC
6)Beta or just hCG: choriocarcinoma, some mixed NSGCTs, and approx 10%seminomas that express it.
7) LDH: Melanoma, Myeloma, Folicular lymphoma, DLBCL
8) Calcitonin: Medullary thyroid ca
9) NSE: can be a marker of SCLC
10) S100: melanoma - not routine at Dx/or ever

How well did you know this?
1
Not at all
2
3
4
5
Perfectly
72
Q

Another high yield non Dose/technique question:

Relate radiation (maximum) doses to fertility and ovarian/testes function:

A

Males:
- Infertility if >6Gy
- Reduced sperm count (and North Queensland quality sperm) 0.15-0.5Gy
- Azoospermia 0.5-6Gy
- Leydig dysfunction 10-15Gy

Ladies:
- Permanent: >12Gy prepubertal, >2Gy near menopause
- Hormone insufficiency 10-15Gy (same as Leydig)

How well did you know this?
1
Not at all
2
3
4
5
Perfectly
73
Q

Timing of adjuvant breast radiotherapy after chemo

A

4-6 weeks

How well did you know this?
1
Not at all
2
3
4
5
Perfectly
74
Q

Dose/approach to endometrial Ca:

State benefit and study

A

All get TAH+BSO+lymph node sampling + washings (peritoneal sampling if serous carcinoma).

Stages:
1A = Observation (includes 1AmPOLEmut)
1B = VBT 28/4 (GOG99) consider ommitting
1C= Pelvic RT alone (45/25, SIB 55 to nodes) GOG249. VBT may be reasonable (PORTEC2)

2-IVa = ChemoRT (cisplatin 5mg D1 and D20) 45/25 pelvis and bed + VBT 10Gy/2#s (PORTEC 3)

Definitive: EBRT 45/25 pelvis,SIB nodes 55Gy, VBT 21/3.+ Chemo.
Or for IA definitive brachy: 27/3

Salvage (if no previous Rad): 45/25 (55 to nodes), 21/3 VBT - Observational data only. + Chemo.

Whole abdo RT for IIIC/V 30/20 is shit compared to chemo alone.

How well did you know this?
1
Not at all
2
3
4
5
Perfectly
75
Q

Broadly, staging (which determines dose) in endometrial ca is based on?

A

Stgs based on: Invasion, extensive LVI, fav (grd I-III) vs aggressive (III, serous, clear cell), local (stg III) regional (III) systemic. POLEmut, p53abn.

How well did you know this?
1
Not at all
2
3
4
5
Perfectly
76
Q

In the setting of endometrial cancer - For VAGINAL BRACHYTHERAPY: Give doses and origin of dose:

A

Definitive: For IA in-operable:
27/3

PORTEC 2 - VBT adj alone. Stage IB
21Gy/ 3, 7Gy/ , 1-2 #/ week
* Prescribed to 5mm from surface of cylinder
* Dose to 5mm cranial from vaginal vault along the axis of cylinder should not vary more than 10% of prescribed dose

PORTEC 3 - ChemoPelvis and VBT if stroma involved.
* 10Gy/ 2, 5Gy/#, 1-2 #/ week
* Prescribed to 5mm from surface of cylinder

How well did you know this?
1
Not at all
2
3
4
5
Perfectly
77
Q

Stg II endometrial Ca what get brachy? outline the technique.

A

Stromal involvement:
Vaginal vault brachytherapy
* Position: supine, use largest vaginal cylinder that will fit comfortably to minimise air pocket
* Target volume: upper half of vagina
* Dose/fractionation: 10Gy/ 2#, 5Gy/#, 2#/ week
* Technique: HDR brachytherapy with Iridium-192 radioactive source, prescribed to 5mm from surface.

How well did you know this?
1
Not at all
2
3
4
5
Perfectly
78
Q

Approach to cranipharyngioma (and dose).

A

If GTR then >90% dont come back = observation.

STR = 70% recurrence = Adj RT = 54/30.

How well did you know this?
1
Not at all
2
3
4
5
Perfectly
79
Q

Indication for adjuvant therapy for heterotopic ossification?
Timing?
Dose/fraction?
Benefit?
Alternate treatment?

A

If recurrent.
<24hrs pre-op, <72hrs post.
Dose fractionation - 7Gy/1# , AP/PA dose to midline, spare a strip of soft tissue.
Reduces risk of recurrence to 10%.
Alternative - Surgery followed by NSAID (indomethacin 75-100mg/day) for 7-14 days post-op

How well did you know this?
1
Not at all
2
3
4
5
Perfectly
80
Q

For pineal tumours what are the indications for CSI

A

After induction carbo-etop (4-6 cycles):
Germinoma (always 1.5Gy#s): Mets = CSI (1.5Gy/#) 18/12 + GTV boost to total 45Gy/30#.
NGGCT (always 1.8Gy#s): Mets or No response to induction CSI (1.8Gy/#) 36Gy/20# (1.8Gy/#) + 18Gy/10# boost to GTV (total dose of 54Gy/30#)

How well did you know this?
1
Not at all
2
3
4
5
Perfectly
81
Q

Approach to pineal tumours (not ependymomas)

A

Always aim GTR.
Pineal parancymal tumours if GTR then obs, if STR consider RT.

Germ cell: InductionCT (carbo etop) x4 if germinoma, x6 NGGCT:
Radiation based on class, and induction response (CT/PR/none) and mets.
1) Germinoma (always 1.5Gy/#, total 30 or 36Gy):
CR: 30Gy total. WVRT 18Gy/12# + GTV boost 12Gy/8#
PR: 36Gy total. WVRT 24Gy/16# + GTV boost 12Gy/8#
Mets: 45Gy. CSI 24Gy/16# + GTV boost to 45Gy/30#

2)NGGCT (always 1.8Gy, total 54Gy)
CR/PR: WVRT 30.6Gy/17# + GTV boost 23.4Gy/13# = 54Gy
No-response or mets:
36Gy/20# (1.8Gy/#) CSI + 18Gy/10# boost to GTV (total dose of 54Gy/30#) + 9Gy/5# to bulky spinal met (total dose of 45Gy/25#)

How well did you know this?
1
Not at all
2
3
4
5
Perfectly
82
Q

Brief epidemiology and associated risks for AVM. How are they graded?
Indication for XRT? Relate to treatment options:

A

1/2000 people. 2-4% chance of bleed/year, At each bleed - 30%chance of morbidity, 10%chance of death. Biggest risk for bleed is a previous bleed. Present from birth, average age become clinically apparent is in 30s.

Graded (Spetzler-Martin) score 1-5: based on Size (<3cm, 3-6,>6cm), location (eloquent/non-eloquent), Presence of deep venous drainage.

Size >4cm attempt emobilisation to facilitate surgery/SRS
Surgery Tx of choice where possible. Improves seizures indicated in S-M score 1-2 (may not be possible in eloquent brain or deep venous drainage).
SRS - tumours <3cm
Observation: may be only option, if previous, aneurysm or deep location should be avoided if possible.

How well did you know this?
1
Not at all
2
3
4
5
Perfectly
83
Q

For AVM give indications dose and expected outcome.

A

SRS for tumours <3cm. with dose 16-20Gy/1# (GTV on angiogram).

Outcome 80-90% at 3years (requires endothelial proliferation to cause obliteration), in latent period risk of bleeding is unchanged.

How well did you know this?
1
Not at all
2
3
4
5
Perfectly
84
Q

Epidemiology of acoustic neuromas:
Preferred approach and why?
XRT dose and outcome:

A

8% Brain tumours, 96% of NF2 get them (often bilateral), 5%NF1 (also oft bilateral). M=F.
Merlin is a Schwan cell tumour suppressor protein (gene on chromo 22), dsPyfunction ->proliferation.

Preferred Mx = surgery, especially in younger Pt - allows debulking and reduction of BS compression (but signif risk hearing loss if tumour >2cm)
Observation=6-12month MR, if growth>2.5mm/year or hearing loss/worsening Sx need Tx.

SRS: 12.5Gy/1# for tumours <3cm and PTV not on BS. Otherwise…..
SRT: 25/5
Outcomes growth arrest (=LC) 85%@3yrs.
It’s possible some crazy big tumours need VMAT 50/25.

How well did you know this?
1
Not at all
2
3
4
5
Perfectly
85
Q

Gamma Knife based brain SRS, ???% is usually chosen as the prescription (marginal) dose.

Typical prescription point for SRS

A

Gamma Knife based brain SRS, 50% is usually chosen as the prescription (marginal) dose.

We (Hamilton) prescribe to 100%, other places prescribe to 60-80%, it’s apparently not an arbitrary choice.

How well did you know this?
1
Not at all
2
3
4
5
Perfectly
86
Q

Dose and technique for adjuvant SRS post resction of a brain met:

Evidence?

A

You could do WBRT (e.g 30/10) isoefective control - but more neurotoxicity and time toxicity (2 phase 3 trials). Neither SRS or WBRT is better than observation in terms of OS.

SRS 12-20Gy ALIANCE Trial
SRT if <30cc 27Gy/3#
If > 30cc 25Gy/5#s

Adj SRT to the surgical cavity, VMAT technique, 6mV photons,
Prescribed to 100% isodose line.

Sim: patient supine, head towards gantry, arms by side, knee rests, comfort measures. Imo: thermoplastic mask.
Contrast CT 1mm fine slice - vertex to C4. Planning MRI T1+C.

HighRiskTumourVolume=HRTV=Surgical cavity
CTV = HRCTV + any GTV + 2mm (may include tract and/or dural margin).
PTV: =GTV +3mm

OARS: in 2Gy/# (I would convert for SRT): Brain stem <45Gy, Brain Dmax<60Gy, V40Gy<5cc.
Verification: daily CBCT matched to bone

How well did you know this?
1
Not at all
2
3
4
5
Perfectly
87
Q

Volumes for adj cavity/brain SRT:

Dose?

When should you be cautious when using SRT?

As an aside what should be done about asymtomatic brain mets (if possible)

A

GTV if any (though if residual present is technically post operative SRS)
High Risk Tumour Volume=HRTV=Surgical cavity
CTV = HRCTV + any GTV + 2mm (may include tract and/or dural margin).
PTV: = CTV +2mm (1-3mm)

DOSE:
30/5 (if >3cm).
OR
27/3 IF<2cm

Caution >5cm, Change to WBRT at about 5.5cm.

Osimertinib only if Exon 19/21 EGFR mutations.

How well did you know this?
1
Not at all
2
3
4
5
Perfectly
88
Q

Options for Liver HCC? SABR dose?

A

No RT or TACE if Childs C (caution in B)
1) Surgery:
-Solitary tumour<2cm do Partial/hemihepatectomy
-2-3 lesions or a single lesion <5cm (Milan criteria) aim liver transplant with bridging therapies while waiting (see below). 5yr OS70% (best chance survival)
2) Ablation:
- Thermal = RFA if <3cm 90%CR, 2-5cm 50-70%, Microwave improved CR for tumours up to 5cm.
- Non Thermal = Irreversible Electroporation: preserves structural integrity of. bile ducts and vessels (used more in pancreas).
3) Trans Arterial Embolisation - +Chemo (TACE) Yttium-90 microsphere (TARE). Local response 50% (TARE up to 70%)
- For lesions too large for TACE (>5cm)
- 85% post-embolisation syndrome 3-4days, 15% irrevesible liver injury.
4) XRT - preferred in patients w/vascular invasion, portal vein thrombosis: SABR 30 or 50 in 5# (LC up to 90%). Conventional 66/33 (French), Palliative whole liver 10/2 for 60%response rate
5) Systemic - no chemo, but Sorafenib increases OS 2-3 months
6) Sorafinib + SABR for disease unsuited to other treatments.

How well did you know this?
1
Not at all
2
3
4
5
Perfectly
89
Q

Role and doses for XRT for HCC:

A

Can be used in bridging prior to transplant.
In combination with other therapies (e.g. TACE+RT reduces recurrence in tumours>3cm).

Preferred in patients w/vascular invasion, portal vein thrombosis. Some retrospective data suggests better LC with SABR compared to RFA,

SABR 30 or 50 in 5# (LC up to 90%).
Conventional 66/33 (French),
Palliative whole liver 10/2 for 60%response rate

In patients unsuitable or refractory to TACE
Sorafinib OS + SABR for disease unsuited to other treatments. SABR adds 3-5 Months OS = 16months.

How well did you know this?
1
Not at all
2
3
4
5
Perfectly
90
Q

SABR doses and volumes for liver HCC

A

Rationale: Equivalent local control to historic RFA (90%<3cm, with better control than RFA for tumours >3cm).

SABR: 30/5 to 50/5 1#/day 5#s week, prescribed to 100% isodose.

4DCT Contrast CT 1mm slice (HCC enhances on arterial phase only). + MRI

Volumes contoured on MIP and checked across all phases of respiratory cycle.
iGTV= All parenchymal (iGTVp) + vascular (iGTVv) on imaging.
CTV=GTV
PTV=CTV+5mm

Motion management system.
Mean liver dose <28Gy

How well did you know this?
1
Not at all
2
3
4
5
Perfectly
91
Q

In what curative setting would you use RT for cholangiocarcinoma? What is the benefit?

Dose?

A

Phase 2 SWOG trial for T2-T4 node positive resected Ro or R1:
Adjuvant RT increases OS compared to historic data (Median OS=3yrs).
45/25 nodes
54/30 to bed
59/33 to R1.

How well did you know this?
1
Not at all
2
3
4
5
Perfectly
92
Q

Approach to Inflammatory breast cancer:

5yr OS

A

NeoAdj chemo -> Mastectomy +ALND -> Adj RT + Receptor targeted therapy

You need pre-treatment clinical photos.
There will never be an RCT, but large retrospective studies demonstrate control benefit of escalation>60Gy.
MDACC (largest study): 66/44, 1.5Gy/#BD, CTV 3cm beyond scar + Bolus until skin rxn too significant. Do this for age<45 or residual post chemo, otherwise can de-escalate - e.g. 60Gy.
5yr OS =40%

If no residual post

How well did you know this?
1
Not at all
2
3
4
5
Perfectly
93
Q

28F w/ Left inflammatory breast cancer, has neoAdj Chemo, mastectomy and ALND. 2cm CW residual, and 14/16 nodes.

What do you do (dose, volume)?

A

MDACC suports dose escalation in this patient (up to 66Gy).

I would treat chestwall and undissected nodes including IMC to 50Gy/25, then boost chest wall to 16Gy. VMAT, DIBH.

Bolus as long as tolerated.

CTV50= GTV+1cm (trimmed to boundaries) + whole chest wall and nodes (level III, SCF, IMC)
PTV=CTV+5mm
CTVboost= GTV+1cm (trimmed to boundaries) + whole chest wall

How well did you know this?
1
Not at all
2
3
4
5
Perfectly
94
Q

Besides chemo, Surgery and RT, What adjuvant treatments need to be considered for pre-menopausal women with receptor positive breast cancer?

A

The SOFT-Text trial:
Ovarian suppression (for 5 years, GnRH agonist=goserelin) added to either AI or tamoxifen improves DFS and OS.

In particular women <35, T>2cm, or grd III, where an AI is preferred to Tamox due to improved DFS and decreased distant failure.

Begin Ovarian suppression at start of chemo

95
Q

For limited stage SCLC Describe a suitable radiation therapy technique and discussion of organ at risk tolerances.

What’s your CTV expansion on the primary?

The chemo?

Also Outcomes

A

5yr OS = 30%
Curative intent EBRT 45Gy/30#s, 2#s/day, at least 6 hours apart, 10#s per week. VMAT technique, 6MV photons, prescribed to D50. Concurrent with chemotherapy = Carboplatin + Etopside.

iCTV=iGTVp + 7mm, iGTVn+5mm
PTV+7mm

96
Q

OARS and tolerances for chest XRT

A

LUNG: Mean lung dose <20Gy, V20gy<30%
Heart: MHD<20Gy,
Spinal cord DMax < 50Gy
Esophagus Mean< 25Gy, Dmx<66Gy
Brachial plexus DMax<66Gy

97
Q

Discuss the effects of ionising radiation on the foetus in the first trimester, and how these are known.

What are the maligancy, retard and threshold risks?

A

Quick numbers. Malignancy 6% increase in absolute risk/Gy. IQ 25 points/Gy threshold hold dose for effects is 0.1Gy:

During the organogenesis phase of foetal development the foetus is at most risk from radiation. Studies of atomic bomb survivors, and other disaster exposures, combined with animal studies demonstrate:
A threshold dose of 0.1Gy
W/general features of Interuterine growth restriction, Cognitive impairment, and future malignancy (6% increase absolute risk/Gy) risk being proportional to dose and also the phase of development at which exposure occurred:
Exposure during pre-implantation phase (day 0-10) - often fatal/failed implantation.
Organogenesis phase (up to 6 weeks): Highest risk of congenital abnormalities, especially CNS (microencephaly). Severe intrauterine growth restriction (due to cellular depletion).
Foetal period 6 weeks to 9 months: 1) Mental retardation - 25 IQ points per Gy, threshold 0.1Gy, due to failure of neurons to migrate peripherally. 2) Microcephaly 3) Eye, skeletal, genital abnormalities 4) Intrauterine growth restriction

98
Q

For salivary gland tumours, indications for XRT

Approach to nodes

A

Benign (e.g. pleomorphic adenoma): Controversial - doses up to 60Gy

Malignant:
Low grade (e.g. Maligant mucoepithelial carcinoma): Surgery and observation.

High grade PORT: 63/30 to R1, 60/30 to R0. 54Gy to nodes (1B II and III).
All adenoid cystic carcinomas.
Close margins (<2mm)
PNI
T3-T4
Node +ve/ENE

ENI:
All HG - But dont bother with ACC
N0 = cover IB-III
N+ = cover 1B, II-V

PNI:
Treat to BOS if: cPNI, rPNI, named nerve, extensive PNI, or adenoid cystic

99
Q

High-grade salivary cancer PORT dose:

A

63/30 to R1, 60/30 to R0. 54Gy to nodes (1B II and III, Unless node + then do 1B to V).

Exception ACC don’t do nodes unless involved nodes (very rare).

100
Q

Indications for thyroid EBRT and doses (and broadly, the targets)

A

Generally age>45.
ENI: II-VII
MTC: If extensive nodes, residual or extension beyond the thyroid.
PTC/FTC: Poor I-131 (e.g. Hurtle cell), I-131 failure, >T3b (Extra thyroid/capsular extension), Gross residual post op.

Anaplastic: Always do PORT, or start 70Gy ASAP if inoperable. Median OS 3months, 5yr OS 10%.

70Gy (or 66/30 to gross residual)
66/30 - to residual/gross
60

101
Q

Dose for Merkels:
And principles of volumes

Potential immunotherapy?

Role of Chemo?

A

With RT - LC 75%, 5yr OS 50% (remember often elderly, immunesupressed).

Definitive: 66/33
PORT:
T1 (<2cm) dont need nodes (but do need LNBx). 50/25+3cm margin.
>=T2:

R0 then 50/25 to BOTH bed +3cm and nodes.
R1 then 60/30 to Bed and 54/30 to nodes.

Avelumab is investigation Stg I-III

Chemo: Consider in stg >I, But no clear role, carboplatin as radiosensitiser.

102
Q

Your exam doses for the Head and Neck sites PORT and Definitive

A

Definitive Naso: 3 dose clouds SIB in 35#s: 70GTV+5mm, 63Gy HR, 56Gy nodes II-V (include IB if node+ve). I.e clouds drop by 7Gy.

Definitive: 70/35 GTV+5mm
63/35 = HR = equivocal nodes, Oral = entire BOT if tongue involved. Oropharynx = also entire BOT if tongue involved. Larynx = entire larynx. Hypo pharynx = entire hypopharynx.

Glottic T1 (limited to vocal cords): RCT supports 55/20 for T1, with concurrent chemo improving preservation. 80% 10yrs. >T1, as above.

PORT: in 30#s, 63Gy (if +ve margin/ECE, historic data from the Peters Trial for ECE), 60Gy to bed and +ve nodes, 57Gy to dissected neck, 54Gy to undissected neck.

103
Q

Evidence and roll of adding chemo to RT for H&N PORT

A

Pooled analysis of 2 RCT demonstrates OS benefit IFF ENE or +ve margin. Interestingly no clear benefit for +Nodes even >5+,

104
Q

Endometrial cancer: What are the indications for vaginal brachytherapy in the post-operative setting? (2 marks) .

A

Curative intent stage 1A: Brachytherapy alone (not favoured approach) for inoperative/declined operative intervention. 27Gy/3#
As part (adjuvant post surgery) curative intent treatment.
1) Stage 1B (>50% myometrial invasion, no LVSI, limited to uterus), without pelvic EBRT. E.g. 21Gy/3. (PORTEC2)
2) Stage II (extension beyond uterus where stromal invasion has occurred) in addition (brachy boost) to pelvic EBRT. 10Gy/2#
3) Stage III and IVa. In addition (brachy boost) to pelvic EBRT. e.g. 10Gy/2# (PORTEC3).

105
Q

ii. Discuss the use of chemotherapy in uterine cancer including in your answer the rationale for its use. (2)

A

RCT Data (PORTEC3) demonstrates at 5years concurrent chemo (weekly cisplatin) followed by adjuvant x4 cycles (Carboplatin Paclitaxel) can provide LC, FFS and OS benefit when concurrent with radiotherapy. It may be considered from grd 3 stg1.
Sub-analysis of PORTEC3 shows the benefit of chemo is primarily in any of the following: Stage III (serosa or nodes involved), Papillary Serous or p53abn.

106
Q

Adjuvant brachy alone for endometrial cancer - give dose and technique:

A

Assume stage 1B: Adjuvant vaginal vault Brachytherapy (Ir-192 source, remote after loader technique), 21Gy/3# to the upper 1/2 of vaginal vault prescribed to 5mm from cylinder surface.

Pre sim: Full discussion of procedure for consent. Analgesia, comfort measures as appropriate. Dilator education with educator (to begin 4 weeks post).

Sim: Patient in lithotomy position. Manual examination and selection of appropriate vaginal cylinder/cylinder segments. Aim largest size that is comfortable to minimise air gaps.

Plan selected from Plan library for applicator selected. Planning system to provide dwell times for after loader.

OARS: Rectum, Bladder.
Verification: External measurement from labia.

Dose Delivery (in specialised bunker with video/audio monitoring, interlocks) via remote after loader.

107
Q

Discuss the role of systemic therapy in the management of metastatic prostate cancer. Include in your answer the class of agents used, duration of therapy and justification for use.

A

Anti-androgen medications - Typically non steroidal anti-androgens (e.g. Bicalutamide): In hormone naive pts: part of initial therapy + GnRH agonist +/- abiraterone (e.g.STAMPEDE) increases progression free survival, and OS. Alternatively can be used for 3-4 weeks as part of commencement of GnRH agonist - to prevent testosterone spike from initial LH surge. Can also be used when progression despite GnRH agonist (“total androgen blockade) to prolong PFS. ADT+ Apalutamide superior DMFS (SPARTAN). Used life-long or until significant progression.

GnRH agonist (e.g Goserelin), provides an PFS, OS and Sx benefit in patients with newly Dx metastatic prostate cancer, and is part of multimodal therapy as above. Duration life-long, or until progression, or in the setting of stable disease intermittent (restart when >10ng/ml).

Androgen synthesis inhibitor - Abiraterone: Improves OS and PFS (STOPCAT RCT)- used in the following contexts - as initial therapy for newly Dx metastatic disease (in combination with ADT but not ADT+Enzalutimide), or when progression on ADT.

Chemo - Taxane chemotherapy - Docetaxel, improves PFS and OS (CHAARTED phase III RCT OS increase from 48 to 58months) when part of initial Mx of metastatic disease. Also increases OS for N+ and locally advanced (STAMPEDE). 75mg/m.sqr (65mg for Asians) until response (up to 10 cycles).

108
Q

In NeoAdj rectum, When may long-course be considered over SC

A

Traditionally, LC-CRT was preferred over SCRT in patients with locally advanced tumours whereby ‘downstaging’ is required to minimise the chance of a positive circumferential resection margin or maximise sphincter preservation.

The lesser ‘downstaging’ effect of SCRT may be overcome with a longer delay to surgery or addition of pre-operative chemotherapy (i.eTNT approach).

109
Q

For rectal cancer: Neo-Adj long course RT is concurrent with?

What is the evidence for chemo?

A

Capecitabine 825mg/M.sqr BD, or continuous 5-FU daily (better than bolus), dont 5-FU dMMR/MSI patients.

Studies from the 1980 and 90s suggested a LC and OS benefit to chemo in post op setting, more significant in the concurrent setting. In the the neoadjuvant setting concurrent improves LC compared to LCRT alone. Recent trials have demonstrated that total neoadjuvant therapy may avoid TME (OPRA - 54Gy+cap -> FOLFOX/CAPOX, or FOLFOX/Capox->54Gy, radiation before better trend) can achieve 50% 5yr preservation rates, w/no change in 5yr DFS = 75% = historical control.

Recently PROSPECT RCT trial demonstrated that neoAdj RT could be avoided (if >20% response on MRI to FOLFOXx6 AND followed by TME and more FOLFOX).

110
Q

A 45yro patient is clinically staged as having an adenocarcinoma of the mid rectum with tumour invading the prostate gland. Multiple mesorectal lymph nodes are present. (cT4N2M0). A decision is made to proceed with pre-operative, long course chemo-radiotherapy to the pelvis. Dose and technique including Chemo & Sim. (not volumes) only:

A

Neoadj long-course chemoRT, EBRT to a maximum of 50Gy/25#, 2Gy/#, 5#s/week. VMAT SIB technique, >=6MV photons, prescribed to D50. Concurrent with oral capecitabine 825mg/m.sqr BD.

Pre-sim: consider if defunctioning indicated, fertility counseling, analgesia, anti-emetics. Comfortably full bladder, bowels empty (enema) prior to sim and each treatment.

111
Q

A 45yro patient is clinically staged as having an adenocarcinoma of the mid rectum with tumour invading the prostate gland. Multiple mesorectal lymph nodes are present. (cT4N2M0). A decision is made to proceed with pre-operative, long course chemo-radiotherapy to the pelvis.

Volumes
OARS

A

Volumes:
GTVp= GTV on imaging and endoscopy
GTVn= Positive nodes on imaging.
CTVp50= GTV+1cm craniocaudally, + mesorectum and pre-sacral space at these axial levels.
CTVn50 (if outside above volume) = GTVn +5mm.
CTV45=CTV50 p and n, Entire mesorectum (+1cm anterior internal margin for bladder filling) and presacral space, external iliacs (as prostate involved), obturators, and internal iliacs to bifurcation. Entire prostate.
PTV=CTV45+1cm

OARS:
Bladder V40Gy<40%, DMax <50Gy (ideally)
Small bowel DMax<50Gy
Large bowel V45Gy<195cc
Femoral heads V40Gy<40%

112
Q

For rectal cancer volumes:
1. Give the T3NO volumes.
2. Invading anal canal (same as?)
3. Invading prostate, upper vag.

A
  1. CTV50 = GTV+1cm craniocaudal, entire mesorectum and presacral space at those axial levels. CTV45= CTV50 + entire mesorectum, presacral space and internal iliacs. 1cm anterior expansion to account for variable bladder filing.

2) CTV50 = GTV+1cm craniocaudal + entire anal canal including sphincters and mesorectum and presacral space at those levels. CTV45=CTV50 NOW include inguinal and external iliacs. (same with lower 1/3 vag).

3) As above. CTV45 = as above, BUT include prostate/vag. External iliacs and obturators. If bladder, may reasonable to do entire bladder (in which case sim empty).

113
Q

A 44-year-old woman presents with vaginal bleeding. Clinical examination reveals a 5 cm ulcerative mass in the lower third of the vagina. Biopsy confirms a primary vaginal squamous cell carcinoma with no evidence of nodal or distant spread. The decision is made to treat the patient with curative intent. The first phase involves external beam radiotherapy with concurrent chemotherapy. Describe a suitable radiation Dose and Pre sim and sim:

A

Curative intent EBRT to a total of 50Gy/25#, 2Gy/#. 5#s/week (see boost below). VMAT Technique, photons >= 6MV, prescribed to D50. Concurrent with weekly cisplatin 40mg/Msqr. Combined with brachytherapy (described below) in final week(s) of treatment.

Pre-sim: Preg test, offer fertility counselling/service referral, analgesia, anti-emetics as required. Comfortably full bladder (also prior to each fraction).

Sim: Supine, arms on chest, head towards gantry. Immobilization with vac bag, knee and ankle supports as needed. Bowel empty (enema at sim and prior to daily RT).
2mm slice CT+contrast, L4-mid femur. Fused with MRI and diagnostic PET.

114
Q

A 44-year-old woman presents with vaginal bleeding. Clinical examination reveals a 5 cm ulcerative mass in the lower third of the vagina. Biopsy confirms a primary vaginal squamous cell carcinoma with no evidence of nodal or distant spread. The decision is made to treat the patient with curative intent. The first phase involves external beam radiotherapy with concurrent chemotherapy. Describe Volumes and OARS

A

Volumes:
For non-nodal volume contour on full and empty bladder and merge CTVs for ITV.
GTV = Primary tumour on imaging and examination.
iCTVp= GTV+5mm, Entire vaginal canal, and Cervix
PTVp = iCTVp+7mm
CTVn = Inguinal, external and internal iliacs (to bifurcation), obturator and presacral nodes (Pre sacral nodes if cervix involved).
PTVn = CTVn+7mm.

OARS:
Bladder V40Gy<40%
Rectum V40Gy<60%
Hips V40Gy<40%

115
Q

For rectum, SHORT COURSE not appropriate in the following:

A

Patient - Sphincter preservation is considered important
Tumour
- cT4 tumour
- N2
- Low rectal tumour (e.g. <5cm from anal verge)
- Presence of EMVI (on MRI)
- Threatened/ close/ positive CRM
- Recurrent rectal cancer (even in patients without prior RT)
Treatment - As adjuvant treatment
- Re-irradiation in patient with previous RT
- When non-operative approach is considered

116
Q

Re-irradiation for rectal Ca (e.g. previuos 50Gy)

A

Retrospective and prospective data suggest safe and associated with better outcomes.

Peter Mac 39.4/20.
I would do: 45/25 concurrent with capecitabine or 5-FU (no if dMMR).

117
Q

Discuss the advantages and disadvantages of the different treatment strategies available for locoregionally advanced laryngeal carcinoma (Stages III and IV). (5 marks)

A

Surgery:
1) For stage III (T3 where thyroid cart not fully penetrated) - Larynx sparing surgery w/followed by PORT (no chemo). Advantage: Avoids chemo, equivalent to definitive ChemoRT (i.e VA study). Histology on resection provides more fine grained prognostic information/risk qualification and decisions on adjuvant therapies (e.g. the addition of chemo). Disadvantages: Worse reported QOL outcomes than definitive chemoRT, operative risks including failed organ preservation and dyspnonia/nerve injury. Does not avoid radiation.
2) For stage IV: Total laryngectomy and adjuvant PORT +/- Chemo. Advantage: Definitive operation, with en bloc resection and detailed pathological information, preferred option for advanced disease. Disadvantages: Offers no chance of organ preservation, highly morbid surgery with poor QOL, intraoperative and post operative risks and complications, despite intensity of intervention may not be curative.

ChemoRT (note the role of induction chemo, e.g. VCF is controversial in terms of benefit vs toxicity, may consider for bulky disease where definitive chemoRT is treatment of choice).
1) Definitive For Stage III - 70Gy/35# with cisplatin (eg. 100mg/msqr)
2) as for 1, potential proceeded by induction chemo.

Palliative:
1) Palliative radiation for local control and symptom MX (e.g. 20/5 or 30/10): Not curative (main disadvantage), can offer good durable local control and symptom relief.
2) In severe case IV - Best supportive care, with medical Mx of Sx, and social emotional and family support.

118
Q

For each of these OARS for pelvis (anal, cervix)
Small bowel loop
Large Bowel loops
Rectum
Bladder
External genitalia
Femoral head/neck
Kidneys

A

Small bowel loop V30Gy<200c, Dmax<50Gy
Large Bowel loops = same as small bowel.
Rectum V40Gy <60%(e.g. cervix)
Bladder: V40Gy< 30% (or 35%), V50Gy<5%
External genitalia: V40Gy< 5%
Femoral head/neck: V30Gy< 50%
Kidneys: Mean dose <15Gy

119
Q

Indications: adjuvant nodal Tx Melanoma

A

Adjuvant treatment (nodal) – TROG 0201
* ≥1 parotid node, ≥ 2 cervical or axillary nodes, ≥3 inguinal nodes
* Largest node ≥3cm (cervical nodes), or ≥4cm (axillary/ inguinal nodes)
* ENE

120
Q

Evidence for nodal irradiation in pelvic node+ve Pr cancer

A

STAMPEDE - FFS benefit 85% vs 55% (compared to no irradiation). This is consistent witrh a number of prospective trials suggestive of significant benefit - including OS.

Dose: 74/37 to prostate and SV + positive nodes. 55/37 to prostate.

121
Q

Give Degarelix MOA

Degerelix vs conventional ADT?

A

Gonadotropin Releasing Hormone ANTAGONIST (as oppsed to the typical GnRH agonsist goserelin/leuporelin).

Phase III open label data demonstrated superior PSA response, and mixed difference in toxicity. Degarelix has more injection site reactions.

Monthly S/c Dose: Dose 1 =240mg. Dose 2-infinity = 80mg.

122
Q

OARS for prostate and nodes plan:

A

OARS (in EQD2): Rectum V40Gy<35% (ideal) may accept up to V50Gy<50%
Bladder V40Gy<30%, Small bowel V45<195cc, femoral heads V40Gy<40%. Large bowel - avoid hotspots.

123
Q

Node positive prostate cancer dose and technique:

What’s a key rule for your nodal volume?

A

Curative intent EBRT with ADT (2-3 years) to maximum 60Gy/20#, 3Gy/#, 5#s/week. VMAT SIB technique, >=6MV photons, prescribed to D50. Aim PGTV D95>98% prescribed dose, avoid plan hotspots over OARS (especially bowel), Dmax<107%.
Pre sim: LUTSxt assessment and Mx as appropriate/if needed. Pre sim and each # - bowels empty (enema) and comfortably full bladder.
Sim: Supine, hands on chest, knee/ankle supports, immob w/vac bag.
22mm slice CT+ con(L3 - upper 1/3rd femur), consider fusion with dx MRI/PSMA PET if +ve nodes small/reduced with ADT.

Volumes:
CTV60p = Entire prostate + 1st 1cm of SVs
GTVn = + nodes
CTV60n= GTVn+3mm (depending on prox to bowel may reduce to 55/20).
CTV44= CTV60 + remaining SVs, pre-sacral (always forget to write that) external iliacs+obturator nodes, internal and common iliac nodes to bifurcation or 2cm above highest positive node (whichever is higher).!!!!!!!!!!!1
PTV= CTV44+7mm.
OARS (in EQD2): Rectum V40Gy<35% (ideal) may accept up to V50Gy<50%

Bladder V40Gy<30%, Small bowel V45<195cc, femoral heads V40Gy<40%. Large bowel - avoid hotspots

124
Q

Evidence for SABR for liver mets:

Give dose and volume.

A

No phase III data (i.e comparing SABR to RFA). Multiple retrospective and 2 phase II prospective trials.

A HyTEC analysis of 13 studies found 1year LCR to be 90%. Outcomes were significantly better for lesions treated with BEDs exceeding 100 Gy (3-year local control 93%).

Rule et. al “Rule” for dose response: LC rate at 2yrs - 100% at 60Gy years, 89% for the 50 Gy cohort, and 56% for 30 Gy.

54/3#, SABR with motion Mx, iGTV to 5mm PTV expansion (if GTV unclear consider 3mm CTV expansion). Contour on CT+contrast - late arterial phase if technically feasible with 4DCT

125
Q

In the treatment of thyroid eye disease with radiation therapy; what are the: (3)
i. indications
ii. contraindications
iii. target volumes
iv. radiation dose fractionation schedules and v. expected response rates to therapy?

A

i. indications
- Clinical exophthalmus due to thyroid eye disease
- Failed trial of steroids
- Not suited to or declined other approaches (surgical decompression)
ii. contraindications
- Responsive to steroids/other Mx.
- Connective tissue diseases (cautionary)
- Eye diseases: e.g. glaucoma
iii. target volumes
CTV= preorbital tissues, PTV 3mm. E.g. with 5 × 5 cm lateral fields using 6 MV photons and 5° poste- rior tilt or half-beam block
iv. radiation dose fractionation schedules and v. expected response rates to therapy?
20Gy/10#, recent evidence suggests that this dose may be reduced (Don’t ever hypofractionate benign). Local control in approximately 2/3rds.

126
Q

For Keloids:
Indications
Dose
Timing
Outcomes
Other options

A

Local recurrence after surgery alone =50%. (25% post RT)

21 Gy/3 fx for most locations and
18 Gy/3 fx for earlobe
24–72 hrs after surgical excision.

37.5 Gy/5 fx if RT is used definitively.

LC 75%.

Other options include steroid injection, cryotherapy, pulsed-dye laser, interferon, or topical agents.

127
Q

Following surgery for cutaneous Squamous Cell Carcinoma (SCC) in the Head and Neck area, what high-risk factors would you consider when recommending adjuvant radiation therapy. (2)

A

Absolute indications =
T4,
+ve margin where further resection not available.
PNI in large nerve.
Recurrence where no previous RT given.

Relative (if 2 or more then do):
T21, 2 or T31.
≥6 mm depth of invasion.
Close margins < 2 mm.
Poorly differentiated.
Pathological microscopic perineural invasion (PNI).
Lymphovascular invasion.
Immunosuppression.
Higher risk sites, e.g. non-hair bearing lip, chin, ear, preauricular, periorbital, temple.

128
Q

Clinically the patient has a 12mm vertical scar in the midline lower eyelid. There is no obvious residual BCC. A decision is made to offer adjuvant radiation therapy.
b. Describe an appropriate radiation therapy technique and dose fractionation schedule. (3)

A

Adjuvant RT to reduce risk of recurrence. SXR technique, 40Gy/15#, 5#s/week, 100% prescribed dose at skin surface, energy/filter/SSD selected for D90 cover at target depth (e.g. 3mm).

Pre: Explain vision changes associated with topical anaesthetic (may need to arrange transport etc).
Sim: Mark-up scar, make or select lead cutout and applicator appropriate to markup + 10mm margin (for CTV + setup error and penumbra). Make setup template from above + setup photos. Eyeshield fitted. Using departmental tables, select Energy/Filter/SSD e.g 20cmSSD 2mmAl, 100Kv for D90 ~3mm (consider contribution of scatter from eye shield to dose). Clinical setup photos.
Volume: as above, Dmax at skin surface aim D90 based on path report.
OARS (in EQD2, note SXR higher BED): Eye: lens DMax<8Gy, Eye <45Gy, lacrimal duct Dmax<30Gy.
Verification via template match and setup photographs.

129
Q

Basic treatment principles for Rhabdo:
- Sequence
- Key variables for decisions
- Dose options
- Which sites inoperable?

A

COGS/ IRS (US/ Australia)
Surgery → chemotherapy → RT

SIOP
Surgery → chemotherapy → no adjuvant RT
* 50% will relapse – intensive therapy for relapse including exenteration + RT (reserve RT for salvage)

Variables (that define risk): Site, Histo. Surgical/Post-op.

Doses:
Embryonal R0 - No Rt
R0 Alveolar and all R1 - 36/20
N+ 41.4/23
Gross disease 50.4/28

130
Q

A 4-year-old girl presents with proptosis. A right orbital mass is identified on MRI. Biopsy confirms an embryonal rhabdomyosarcoma. Further staging investigations show no metastatic disease.

Give dose and technique.

A

Definitive EBRT 50.4Gy/28#, 1.8Gy/#, 5#s/week to the right orbital mass with concurrent chemotherapy (vincristine and cyclophosphamide; and omitting actinomycin-D during RT)
Pre-SIM:
* Repeat MRI post chemotherapy
* Consider GA if patient not able to remain still on RT SIM
* Position: supine, arms down, head towards gantry, under GA
* Immobilisation: thermoplastic mask, vac bag, neck support,
* Marker: no specific
* Planning CT: 2mm slice CT from vertex to lower cervical spine Fusion: pre-chemo and post-chemo MRI
Targe volume:
* GTV = pre-chemo gross disease
* CTV = GTV + 1cm clipped at anatomical (i.e. bone) boundaries (no need to cover entire orbit)
* PTV=CTV+5mm
Technique: partial arc unilateral VMAT technique with 6MV photon Plan evaluation:
* Tumour coverage PTV D98>95%
* Minimise hotspot PTV D2<107%, ensure no hotspot outside of PTV/ over critical OAR
* Ensure low dose (10% and 50%) reasonably distributed around PTV, with contralateral sparing
* Review OAR DVH
o Pituitary and temporal lobe ALARA
o Lacrimal gland Dmax< 41.4Gy
o Optic nerve/ chiasm/ brain stem: Dmax< 50.4Gy o Lens Dmax< 14Gy
Verification: Daily kV, matched to bone, with 3mm tolerance

131
Q

Immunotherapies (name targets and mechanism) for BRAF negative melanoma:

What is the benefit?

A

If no BRAF than target T-Cell mediated cell death evasion:

CTLA4 - Ipilumimab. Normal role is to downregulate T-Cell activation. Upregulated in cancer leading to immuncheckpoint evasion. Ipilumimab binds CTLA4 and inactivates it.
By itself adds 3 months PFS.

PD-1 - E.g. Nivolumab. Binds PD1 receptor on T-Cell preventing PDL-1 from binding and inactivating T-Cell,
By itself adds 6Months

Ipi + Nivo = 12months PFS

OS for metastatic BRAF negative is 45%

132
Q

Immunotherapies (name targets and mechanism) for BRAF positive melanoma:

What is the benefit?

A

Best results combine BRAF and Mek targeted therapies:
TKI signal transduction pathway RAS RAF MEK ERK

Braf: Dabrafinib

MEKtargeted by molecular inhibition = Trametinib.

Get on the ‘ol Dab Tram for 1yr OS 70% next stop death.

133
Q

Doses for meningioma:

A

If optic nerve or being gentle - 50.4/28

Grd 1
R0 -observation
Simpson 1-3 - Observation versus 54/30
STR/primary Mx - 54/30
SRS 14/1
SRT - 25/5

Grd II/III
For grd II Observation is investigational (ROAM)
Simpson I-III 54 to 60/30 or 59.4/33 (if big or young!!!)
STR/residual - 60/30 or 59.4
SRS/SRT - 14G or 25/5

Note: 59.4Gy/33, 1.8 Gy# where large volume or young patient 1.8Gy/# - a common Exam trick question.

Grd I-III Recurrence: Re-resection where possible
Then doses as per relevant grade

134
Q

Volumes for menigioma:

A

GTV = gross disease (T1 + contrast) [if no resection]
GTV(post-op) = residual disease (post-op MRI) (T1 + contrast)

Grade 1
- CTV = GTV + 0.5cm for WHO Grade, 5mm into brain, clipped at anatomical boundaries
Grade 2
- CTV = GTV + 1 - 1.5cm along the dural for WHO Grade 2/3, 5mm into brain, clipped at boundaries

PTV=CTV+5mm

135
Q

pathology demonstrates a WHO Grade 3 (anaplastic) meningioma. Post-operative imaging shows residual disease on the floor of the anterior cranial fossa. A decision is made to give post-operative radiation therapy.

b. In this woman, describe the GTV, CTV, and PTV you would apply and explain how would you determine these?
(2m)

c. What dose would you prescribe? Justify your answer (1m)

A

Fuse pre-op and post-op MRI
GTVpost-op = GTR based on fusion of PO MRI
CTV = GTV + surgical cavity + 1.5cm along dural + 5mm into brain, clipped at anatomical boundaries PTV = CTV + 5mm expansion

I will offer her adjuvant RT to a total dose of 59.4Gy/33, 1.8Gy#, 5#/ week
- Large volume hence use more fractionated + young patient 1.8Gy/# (lower late effects than 2Gy/#)

136
Q

Prostate:
Is there a benefit to elective pelvic nodal RT (ENI) and which patients should be considered?

A

Historically trial data has been unclear or not supportive (e.g. GETUG, RTOG). More recently the POP RT trial, which studied a population of higher-risk of nodal involvement (>=20% by Roach) demonstrated a 5yr met free, disease free benefit but with higher late GU toxicity. WPRT = 90% 5yr disease free survival vs 75%. This study used a modern prostate dose (68/25), and 80% of pts had a PET.

A new RTOG study 0924 is recruiting with an aim for higher-power than previous.

137
Q

Which non-high risk prostate Ca patients should get ADT? Give duration and evidence.

A

Int risk Unfav - ADT 6mths = DFS, biochemical failure reduced from 20 to 10% at 5yrs. PrSpec survival and potential OS benefit:
NCCN recommendation is for ADT beginning at intermediate unfavourable prostate cancer. Its is based on 2 phase 3 RTOG trials and meta-analysis. The 1st RTOG trial 9408 found 4 months ADT provided a DFS benefit to IR-U but not IR-F (on subanalys).

A 2nd RTOG phase III trial found int risk PrCa in general benefit from 6 months ADT in terms of DMS and prostate specific survival.

Meta-anlysis suggests a small OS benefit at around 9 years in int prostate cancer.

Notably ADT effect is independent of radiation dose.

138
Q

What is the optimal duration of hormone therapy for HR prostate cancer?

A

OS benefit has been demonstrated with long-term ADT (28–36 months) compared to short-term regimens (4–6 months), even in the dose-escalated era (DART trial).

EORTC trial 3yrs ADT improved 10yr OS from 40%-58%

One trial (PCS IV) found similar oncologic outcomes between 18 months and 36 months.

The RADAR trial found that 18 months of ADT was superior to 6 months prCa spec OS 13vs10%

139
Q

Ignoring more modern trials looking at nodes ect:

In men with a detectable Prostate Specific Antigen (PSA) after prostatectomy for prostate cancer, salvage prostate bed alone yields what benefit?

A

In men with a detectable Prostate Specific Antigen (PSA) after prostatectomy for prostate cancer, salvage prostate bed radiotherapy (PBRT) results in about a

70% freedom from progression (FFP) at 5 years

140
Q

What approach most significantly impacts outcomes in early salvage prostate cancer treatment. State the benefit of this approach.

A

Remember by treating the bed alone you achieve 70% freedom from progression benefit at 5yrs.

The SPPORT trial (3-ARM) shows that with treatment of:
Traditional arm = Bed RT = 71%
Bed +6 months ADT benefit increases to 81% (a 10% gain)
Bed+WPRT+ADT = 87% (16% gain!)

Keep in mind OS at 5yrs is great regardless - 95% so no difference.

141
Q

PRIME II

Criteria
Key findings

A

Age>65, ER or PR +, node -ve, margin >1mm, THEN can have either LVI or grd 3 BUT not both.

10 study.
RT + tamoxifen = Tamoxifen for OS, and DM.
ONLY benefit is a reduction in IBTR from 4% to 1%

142
Q

For prostate cancer:
Early Salvage vs Adjuvant(AKA?)?

Key studies:

A

3 key proRCT studies: RAVES (failed to meet criteria), GETUG-AFU17, RADICAL suggest Salvage is non-inferior to Adjuvant radiotherapy in terms of event free survival - but is associated with less late GU toxicity, and some men may be spared radiation entirely.

Salvage being defined as when PSA crosses a defined point (usually 0.2ng/ml).

143
Q

Dose/prescription for salvage EBRT post RRP:

A

RAVES uses 64/32 to bed,

SPORT (bed+nodes+ 6 months ADT) uses up to 70Gy to bed, 45Gy to nodes.

As we are only treating microscopic disease - I would use 55/20 (2.75Gy/#) to the bed and 44Gy to the nodes. EQD2 68Gy (for prostata alpha/beta 1.1), but only 60Gy in terms of bowel tox, and 58Gy in EQD2 for bladder tox. NCCN supports.

144
Q

For Neuroblastoma give the treatment sequence based on Risk:

A

Low - Risk - 4S - Observation. Spontaneously resolve 5yrOS 95% (4S = age<1, ipsilateral node negative, Age <1y/o with <10%bone marrows, even mets to skin or liver)

Low risk: Surg alone for non-4S = GTR: No benefits for adj RT
* If STR, or recurrence after GTR: consider chemo

Int risk: Induction chemo then surg, RT only if progression.

HR: Always need multimodality treatment
Sequence: Induction chemotherapy → surgery → consolidative chemotherapy → autologous
stem cell transplant → RT (primary + metastatic site + boost residual disease) → differentiation therapy and immunotherapy.

Consolidative RT = 21.6Gy/ 12# (1.8Gy/#) to tumour bed, unresectable bulky sites, metastatic site
o Boost residual disease to total of 36Gy/20#

OS 15% at 5yrs!

145
Q

Define the following:
Absolute Risk Reduction (ARR)
Number needed to treat (NNT)
Relative Risk (RR)
Relative Risk Reduction (RRR)

A

ARR = I Experimental event rate - Control event rate I
NNT = 1/ARR
RR = Experimental Event rate/Control Event Rate
RRR = ARR/Control event rate = 1-RR

146
Q

When delivering post mastectomy radiation to the chest wall:
i. What is the rationale for the use of bolus? (1)

Ii. Which patients require bolus? (2)

A

i. What is the rationale for the use of bolus? (1)
- to reduce dose to underlying lung
- Increase skin and subcutaneous dose (most common site of recurrence post mastectomy) and
- Smooth some post mastectomy tissue irregularities.
Ii. Which patients require bolus? (2)
- Pts with close/involved superficial/skin margin
- Pts with highly irregular surface contour
- Pts with shallow skin to lung interface depth,

147
Q

An 80-year-old man is diagnosed with a 3cm squamous cell cancer near the hilum of his right lung (cT2N0M0). Investigations reveal no other sites of disease.
A decision is made to treat with definitive SBRT using a linear accelerator. Describe a suitable radiation therapy technique and dose fractionation schedule. (4)

A

Curative intent SABR 50Gy/5#s (using central tumour), 1#/day, prescribed to PTV D95>100% prescribed dose, 6MV photons, DCAT technique.
Aim Dmax within PTV 125-14% of prescribed dose, CI-100; 1.1-1.3, CI-50<5.
Presim: Baseline lung function
Sim: Supine arms above head, head towards gantry, knee/ankle rests, head support
Immobilise with vac bag. Patient breathing freely.
4DCT + contrast, fusion with PET if available.

Volumes: iGTV = tumour volume contoured on MIP and checked against all phases of breathing cycle and referenced against PET,
iCTV=GTV, PTV=iCTV+3mm
OARS (in 2Gy#s, would convert or use published data - e.g. from CHISEL): Spinal cord Dmax<45Gy, brachial plexus Dmean<60Gy, Lung V20Gy<10%, oesophagus, heart MHD<2Gy.

148
Q

Locally recurrent nasopharynx Ca.
A decision is made to retreat with radiation therapy. Discuss a suitable external beam radiation non-proton technique. (2.5)

A

Curative intent re-irradiation, hyperfractionated approach 60Gy/40, 2#s/day, 10#s/week, VMAT technique, 6MV photons, prescribed to D50. Aim PTV D98>100% prescribed dose. Avoid Dmax>107%.
Pre-sim: Consent to include discussion of significant risk of potential severe toxities and death
Sim: Supine, head towards gantry, arms by side, knee board, ankle supports. Immobilise with thermoplastic mask.
2mm fine slice CT+contrast, above vertex to T4, and fused with PET and planning MRI (T1+C).
Volumes:
GTV: defined on imaging and endoscopy.
CTV: GTV+5mm trimmed to boundaries,
PTV = CTV+5mm
I would not include elective nodal regions.
Verification: Daily CBCT matched to bone.

149
Q

When treating a NSCLC to 60Gy/30:
What is the expansion on iGTV?

What is the Chemo? Compare that to SCLC

A

SCC - 8mm
Adeno - 6mm

Chem0 = Carbo Etop = both 50mg/msq

SCLC = Carbo Etop = 5AUC!!! + 100mg/msq

150
Q

When would you consider PORT for vulva cancer?

What is the most important prognostic factor?

A

Node status is by far and away the most important - 1 inguinal node 50% 5yr OS, 1 pelvic 11%, node - 90%

The Heaps study was one of the 1st to identify RFs.
Critical are:
Margin <8mm
Depth >5mm
LVSI/VSI
Size>4cm
Stage III = Node +ve (GOG37!! OS benefit when clinical node +ve, more than 1 node, ECE or ulcerated).

Others from Heaps (but not critical to the answer):
Keratin production, mitotic rate.

151
Q

Benefit, evidence and role for Neo Adj Chemo RT for unresectable vulva cancer.

Give dose and volumes.

A

GOG 205 - Phase II unresectable (T3-T4, N+):

Pathological complete response in 50% with this dose escalated regimen:

45/25 to primary + nodes: Nodes and primary boosted to 59.4/33#s.

Essential CTVprimary = GTV+1cm, Subcutaneous Mons bridge and whole vulva.
- Add ons:
Node volumes: If +ve treat bilateral level, treat the eschalon above bilaterally.

Involvement of distal vagina, include:
bilateral inguinofemoral,
bilateral obturator, and
internal and external iliac nodes.

If proximal posterior vaginal involvement, include pre-sacral nodes (S1 to the bottom of S2/top of S3).

If involvement of anal canal, include as per distal vagina and include perirectal, mesorectum and presacral nodes.

152
Q

Dose for PORT vulva:

?Adjuvant ChemoRT

A

+ve margin - advocate re-excision.

Divide into >2cm solitary node negative (i.e Ib-II) :
R1- 54/27, R0 - 50/25.

“Indications” : Margin <8mm (or 5mm…), DOI >5mm, LVSI, lesion>4cm, “Spray” (diffuse) histology,

Node +ve:
Micromet <2mm on Bx can have nodal irradiation instead of dissection

Post inguinal dissection – if >1 node +ve Or “less extensive” dissection (<12 nodes).
50Gy/25 +/- 10Gy boost to positive nodes

?Adjuvant ChemoRT – no prospective data. Some retro analysis suggests possible OS benefit. Extrapolate from Cx Ca and consider when bulky nodes, +ve margin

153
Q

Role of chemo in vulva Ca?

A

?Adjuvant ChemoRT – no prospective data. Some retro analysis suggests possible OS benefit. Extrapolate from Cx Ca
and consider when bulky nodes, +ve margin

154
Q

Volumes for PORT vulva when >1node +ve

A

CTV54p=positive margins and ECE (CTV54n)= Entire surgical bed +2cm
CTV50nodes:
1) Treat echelon above highest involved node bilaterally.
2) If inguinofemoral node involved, treat contralateral inguinofemoral region and external iliac nodes.
3.1) Involvement of distal vagina, include:
- bilateral inguinofemoral,
- bilateral obturator-
- internal and external iliac nodes.
3.2) If proximal posterior vaginal involvement, include pre-sacral nodes (S1 to the bottom of S2/top of S3).

4) Anal canal (same volumes as anal primary), include as per distal vagina:
- bilateral inguinofemoral,
- bilateral obturator-
- internal and external iliac nodes
&
perirectal, mesorectum and presacral nodes.

155
Q

What is pentoclo protocol for osteoradionecrosis?

A

PENTOCLO protocol:
Pentoxifylline 400 mg, BD
Tocopherol 500 mg, BD
Clodronate BD
5 Days a week
Continue til “acceptable clinical response”

156
Q

Approach to PORT (or definitive) nodal irradiation for oral cavity SCC

A

1) Decide if unilateral or bilateral:
- Unilateral only if single ipsilateral node & tumour well lateralised - at least 1cm from midline, BOT or tip of tongue.
- Bilateral if >=2 ipsi nodes, close to midline.

2) Which nodes?
IB, II and III always
Any tumour close to midline or into oropharynx then include level IV. Obviously also if III involved.

If bucal mucosae then level IX (buccal nodes) ipsilateral nodes.

157
Q

Approach to PORT (or definitive) nodal irradiation for Oropharynx SCC p16 negative.

A

Trick - p16+ may also be the same - unlear.
1) Decide if unilateral or bilateral (similar to oral cav):
Unilateral if NO-N2a (single ipsilateral node <6cm).
Tonsil fossa tumor NOT
infiltrating the soft palate OR
the BOT

2) Which nodes?
If ipsilateral then: II, III, IVa’,
+ 1B if oral cavity extension (e.g. into retromolar trigone)
+VIla = retro pharyngeal nodes for posterior pharyngeal wall tumor

If bilateral: Ib, II, III, IVa, Va,b, +VIla,
+VIIb* only do bilateral retropharygeal nodes if posterior paryngeal wall tumour.

158
Q

Definitive dose and volumes for a T1-T2 glottic larynx:

Define T1 and T2.

A

T1 = vocal cords (a = uni, b=bi)
T2= extension in supra/sub glottis.

60/25!!!
CTV= Entire larynx at level of GTV and 0.5-1.0 cm superiorly and inferiorly.

159
Q

Compare volumes for larynx and hypopharynx

A

CTV70 = GTV + 5-10mm both sites.
CTV63 = CTV70+ intermediate risk nodes +:
If Larynx SCC: consider entire larynx
If Hypopharynx: entire hypopharynx

CTV56=Bilateral nodal levels II, III and IV.
Consider levels Ib and/or V included on side(s) of lymphadenopathy.
Consider VIIb and level V for extensive level II lymph node involvement.

IF
Larynx: VI if subglottis

Hypopharynx: VI for apex of piriform sinus, post cricoid and/or oesophageal extension

160
Q

For H&N cancers define:
Positive/involved margin:
Close margin
Clear margin.

Large-calibre and small-calibre nerves

A

Positive/involved margin: <1mm
Close margin: 1-5mm
Clear margin: >5mm

Large-calibre and small-calibre nerves: Large >0.1mm, small <0.1

161
Q

Dose and rationale for SABR to RCC mets.

A

No phase III data, but pooled and meta-analysis suggests LCR>95% at 5yrs (potentially better than historic ablative controls).

Phase II data SABR + pembro offeres excellent local control rates.

Indication is inoperable and not suited to ablative techniques.
(dont give concurrently with molecular therapy!!)

Dose of SABR: 26/1 (smaller lesion <2.5cm), 40/5 (up to 5cm)

162
Q

Briefly give the argument for treating pelvic nodes in node negative prostate cancer:

A

There is no large randomised trial evidence (GETUG-01, RTOG 9413) supporting that PNRT
improves oncological outcomes. A recent smaller phase III trial (POP- RT) showed improved biochemical control with PNRT 50Gy/25.

163
Q

For each of the following simply give doses, the study they cam from, a guideline supporting their use (and the evidence grade assigned by that guideline):

Prostate-only RT:
* EBRT only
* EBRT + Brachy
Stereotactic radiotherapy (SBRT):

Pelvic nodal RT (node -):
*The current gold standard.
*Your approach

Postoperative RT:

A

All the below doses are within UK-RCR
Prostate-only RT:
* 60 Gy/20 over 4 weeks CHIiP Trial (Grade A)

EBRT+Brachy
* 46 Gy in 23 fractions (prostate and pelvic nodes) over 4.5 weeks followed by 15 Gy HDR or 115 Gy LDR brachytherapy boost (Grade A)

SBRT:
* 36.25Gy/5(Grade A) - 2 trials: Hypo RT (42/7), PACE-B 36.25/5 (USE). Same outcomes as conventional, but possibly higher late grd II GU.

Pelvic nodal RT node -ve:
* 50Gy/25 over 5 weeks or equivalent (Grade A) - POP-RT current only trial for node-ve to show benefit - therfore this is current standard.
*44-47/20 are in common use. 46/20 (2.3Gy/#) = 50Gy BED (Grade D)

Node +: STAMPEDE - PFS benefit. 60/20 to Pr, 44/20 nodes, 55-60Gy to +ve nodes (OARS permitting).

Postoperative RT:
* I favour early salvage and treat nodes (SPORT Trial 64.8/36, 45/25 nodes), but dose I prefer 55/20 (60/20 is PSMA residual) and 44/20 to the nodes

164
Q

Name a rectal spacer brand. How many mls of spacer?

Evidence?

A

Barigel - 9 to 10mls.

Results from 2 RCTs were varied; however, the use of rectal spacers showed improvement in rectal toxicity and quality of life at up to 36 months. More severe toxic effects were less common.

165
Q

A decision is made to administer TBI using an opposed lateral technique. Describe this radiation technique and include an appropriate dose fractionation schedule. (5)

A

TBI delivered to a total of 12Gy/6#, 2#s/day treat over 3 days, opposed lateral photons (SSD 4m), aim energies below 6Mv, prescribed to midpoint (typically umbilicus). Alternating between lateral beams at each fraction. Aim for 7 days prior to transplant.

Pre sim: Anti-nausea medication, other meds to consider are dexamethasone and anti-anxiety meds (e.g. Lorazepam). Fertility preservation (though they’re pretty fucked if not done by this point).
Sim: Supine, small under head, knee fix, Upper arm resting on 4cm polystyrene (to reduce lung dose), Hand resting on abdomen.
Compensator/ bolus (‘beam spoiler’) – (because of skin-sparing effect of photon):
Bolus (E.g. flab) – on lateral and anterior surface of neck + chest (to reduce dose to lung)
Perspex – as head frame compensator, and from mid-thigh inferiorly (thicker from mid knee
inferiorly)Linac field typically at most open position/largest practical field size.
Volume: Whole Body – Covered by light field. Aim homogenous dose +/-5% of PD.
OAR: Lungs – Aim <12.6Gy, consider Lung shields.

Verification:
Patient position confirmed with laser matching. Whole body within light field, in vivo dosimetry (e.g. TLDs on skin surface) not routine.

166
Q

Contraindications to bladder preservation approach:

A

Assess for relative contraindications: Tumour>5cm, multifocal/in-situ, incomplete TURBT, irreversible hydronephrosis.

167
Q

What is the expected 5-year overall survival following treatment for muscle invasive bladder cancer?

Ii. Discuss the difficulties that arise when comparing survival outcomes between radical cystectomy and bladder preservation treatment approach.

A

Recent (2023) Pooled analysis of phase II data suggests 5rOS for treated bladder Ca is 75% (slightly higher than surgery despite being a more com-morbid cohort). 66% chance of bladder preservation.

Difficulties when comparing survival outcomes between cystectomy and bladder preservation:
- No direct, randomised trial exists to demonstrate superiority/ (non) inferiority.
- Populations may not be directly comparable - Bladder preservation candidates may have more co-morbidities/tumour complications than those selected for radical surgery.
Comparisons with historic data in terms of patient outcomes may be out of date due to new surgical (increased options for conduit) and radiation (VMAT vs planned) techniques.
Planned technique may have treated nodes!!

168
Q

Indications of breast boost - name who recommends?

Key study and who benefits the most?

A

ASTRO (2018) Guidelines:

50 years or younger with any grade tumor, or in patients aged 51 to 70 years with high-grade tumors or positive margins.

***Consistent w/subgroup analysis of Bartelink (now 20yrs) - suggests greatest benefit age<50. Also those w/adjacent DCIS (at long term follow up) and grdIII

169
Q

Who gets the most benefit to post mastectomy RT? What Study?

Also define N1, N2 and relate to stage.

A

PMRT improves both loco­regional control and OS in both pre and post­-menopausal women w/locally advanced BrCa.

A large EBCTG pooled analysis (which is pre Her-2 era, pre modern LND/Bx and RT techniques) but is the most comprehensive to date:
1) T1-T2 with 1-3 nodes (i.e N1, stage II) - LRR and BrCaS benefit
2) 4 or more nodes (Stage III) - LRR and BrCaS benefit.

SUPREMO is ongoing looking in the modern era at the extent to which this benefit extends to: T2 grdIII, T2 LVSI, T1-2N1.

RCR UK supports 26/5 (Fastforward had 90pts, only recurrence was in 40/15grp) as Grade A.

170
Q

Early Breast:
In general, what are the options for the further management of the axilla where one sentinel node is positive?
Justify your answer (2m)

A

1) Observation (i.e. no further axillary treatment)
Z0011 study, patients with 1-2 SLN +
- Randomised to ALND vs. observation
- no differences in nodal recurrence/ OS with between arms
- higher risk of lymphoedema in patients who had ALND (13% vs. 2%) Also, given that she has endocrine therapy and targeted therapy options
2) Axillary RT
AMAROS study – patients with SLN+
- randomised to level 1-3 ALND vs. axillary RT 50Gy/25#
- no differences in axillary recurrence (primary endpoint) or OS between arms
- less lymphoedema with axillary RT (23% vs. 11%)
3) ALND In AMAROS, ALND arm
- 33% had further LN+, and 8% had 4 LN+
4)Clinical trial POST-NOC (1-2 SLN+, macro mets >2mm)
- randomised to observation vs. axillary treatment (surgery or RT)

171
Q

Describe MA.20 RCT:

Notable inclusion?

A

MA.20 (median follow-up was 9.5 years):
Patient cohort: node+Ve or high-risk node-Ve breast treated w/BCS and adjSystemics:
WBI+ plus RNI (including internal mammary, SCF, and axillary) (nodal-irradiation group)

VS

Whole-breast irradiation alone

Notably, IM nodes were included.

Found: RNI + WBRT did not improve OS but reduced the rate recurrence = 5% improvement in DFS at 10yrs (they talk up the “40% relative risk reduction”)(But in general most pts did well so this number is small - pts w/>3 nodes tended to be excluded as they all had radiation- taking away the strength of the signal).
Higher rates of grdII pneumonitis

172
Q

Your position on IMN radiation (justify):

A

I would treat IMN if N2 (Thorsten prospective), or N1 and medial tumour (KROG).

Major RNI studies (MA.20) have included IMN and demonstrated minimal added toxicity.

The Thorsten prospective trial suggests an OS benefit to IMN (that studied included up to 9nodes+ve) for N+

Krog - DFS benefit in medial N+ tumours

173
Q

32F completely excised left breast IDC tripple pos, grd II T2.
She undergoes a left level II axillary dissection and is found to have a total of 4 macro-metastases from 15 nodes. She is referred for adjuvant radiation therapy.
Provide justification for your choice of nodal volume (1m)

A

This pt with N2 disease following ALND would benefit from RNI to levels 3, SCF and IMN. I would boost the surgical bed for improved local control given age <50 (Barlelink 20yr data, Astro guidelines).

As per MA.20 and EBCTG meta-analysis this is likely to improve DFS (in the order of 5% or more). MA.20 included IMN, and specific benefit is noted in prospective data (Thorsten) when N2 disease.

174
Q

32F completely excised left breast IDC T2.
She undergoes a left level II axillary dissection and is found to have a total of 4 macro-metastases from 15 nodes. She is referred for adjuvant radiation therapy.
d. Describe a suitable radiation therapy technique and dose fractionation schedule. (2m)

A

Adj EBRT for improved IBTR, and DFS to the left breast and level III, SCF and IMC nodes, 40.05Gy/15#, +10Gy boost to the tumour bed, 5#s/week. VMAT 6MV photons, breathhold technique, prescribed to D50.

Pre sim: well-healed, assess for baseline lympoedema, Beta-HCG.
Sim: the usual, DIBH. wire scar. no bolus.
2mm slice CT+contrast.

CTV=all breast tissue on imaging, trimmed 5mm off skin, deep border fascia of pec major. Level III, SCF, and IMN intercostal space 1-3 (5mm expansion on internal mammary vessels trimmed to boundaries).
PTV=CTV+5mm
CTV boost= scar tissue on imaging, encompassing surgical clips
PTVboost = CTVboost+5mm

175
Q

Relate surgery to stage of Wilms and indication for RT, what other criteria relate to this stage?

What other factor is an indication for RT?

What is stage V, how is it conceptualized?

A

Stg III is largely based on surgery events or non-events:
Bx (i.e. causes seeding)
Seeding
LN +Ve
Unresctable
Ruptue
Piecemeal rescetion
B_SLURPP

Unfav Histo (UH) = Diffuse anaplasia = 19.8Gy/11.

Stage V is bilateral kidney disease at Dx - Stage each lesion separately!

176
Q

Simple principles for RT for Wilms:

A

Residual disease gets a boost - 10.8Gy/6# (i.e double the R0 port dose).

Siop Dose is higher

Stg III get at minimum 10.8/6 (i.e. +/- boost)
UH get 19.8/11

NWTS:
Upfront Nephrectomy -> RT if indicated - VACD-IE
Siop:
Upfront CTx (VA) - Delayed nephrectomy - Chemo

Whole Abdo RT (10.5/7) if Spill, Peritoneal Seed, Ascites Cyto+, Rupture (SPAR).

Whole Lung RT 12Gy/8: if lung mets (NWTS), incomplete path response to CTx (SIOP)

177
Q

Doses for Wilms (NWTS)

A

Whole Abdo RT (10.5/7) if Spill, Peritoneal Seed, Ascites Cyto+, Rupture (SPAR).

Whole Lung RT 12Gy/8: if lung mets (NWTS), incomplete path response to CTx (SIOP)

Stg III
10.8/6 to bed.
UH
19.8/11

Boost
10.8/6

178
Q

The “not low risk” DCIS patients who should be considered for a boost:

A

“the big 3” trial is utter bullshit - they changed the definition of risk due to failure to recruit, they dont say, but the NNT is 24 (they sell the benefit as 40% risk reduction! But the control event rate IBTR was 7% the experimental rate 3%). 16Gy extra to the boob for that. Total nonsense.

Age <50

or
Grd II or higher, multifocal, margin<10mm, size >1.4cm, palpable or necrosis (so basically everyone. total BS).

I would boost age <40, grd III, +margin or necrosis.

179
Q

WLE+RT vs Mastectomy (recurrence out comes)

A

Broadly: LRR 10-15% WLE+RT, 2%Mastectomy

180
Q

a. In general, what are the indications for adjuvant radiation therapy in the management of thymoma? (3m)

b.Give doses and iCTV for definitive and adjuvant thymomas.

A

a. In general, what are the indications for adjuvant radiation therapy in the management of thymoma? (3m)
- Stage II-IVA disease (Masaoaka Stage)
- Residual disease (R1-R2)
- Thymic cancer (i.e. WHO Grade C)

b.
Definitive - 60Gy (minimal evidence, but if higher-grade consider concurrent chemo carbo-pac)

R0 - 45/25 (e.g. thymic carcinoma)
R1 - 54/27.
R2 - 60/30

iCTV = iGTV (if any)+5mm, entire thymic space.

181
Q

Post operative soft tissue sarcoma dose and volumes:

A

PORT:
CTV60= pre-op GTV/tumour bed +1.5cm radial (as above) + !2cm! sup inf
CTV50= CTV60+2cm sup-inf
PTV = CTV50+1cm

182
Q

iii. In general, what is the goal and magnitude of the benefit from adjuvant radiation therapy for DCIS.

A

The goal of therapy is to reduce ipsilateral breast tumour recurrence (approximately 50% of recurrences are invasive disease). Broadly (depending on prognostic factors) recurrence at 10 years ranges between 10-30% (very significantly less if hormone therapy part of treatment), 50% or recurrences are with invasive disease. With adjuvant RT reducing by more than half (~60%) this risk.

183
Q

Discuss the role of surgery in liver HCC (include and compare):

A

Surgery - offers highest chance of cure:
- Partial/hemihepatectomy: Indications solitary tumour <2cm. Generally well tolerated, offer highest chance of cure >70%5yr OS, but most disease does not present when indicated. Contraindications= multiple tumours, size>2cm, invasion into vessels. Pt must be fit for surgery.
- Liver transplant: Offers excellent survival (Os 70% 5 years), however has long wait times for suitable donor, and often requires bridging therapy, pt must also be fit enough for intensive surgery.

Indications (UCSF):
Solitary <6.5cm (or 5cm Milan), or up to 3 small lesions with cumulative size <8cm.

184
Q

Approach to a patient with a pacemaker getting RT:

A

2 types: Cardiac output dependent pacing or not dependent.
At booking initiate pacemeker pathway )physic team, planners aware, device contoured).
In general avoid >10Mv if possible, total dose <2Gy.

  • Dependent: Vitals monitoring during each #, Device function assessment during treatment and at end.
  • Independent: If above constraints not met then vitals during. If met then just device assessment at end of Tx.
185
Q

Outline the Hx of low grade glioma trials:

Treatment approach to low grade glioma

A

1) Non-believers: Early adjuvant - better PFS but not OS
2) Believers: 45Gy=59.4Gy, No OS benefit. SATAN criteria: Size>6cm, Age>40, Tumour crosses midline, Astrocytoma, Neurology. 3-5 factors = high risk.
3) Shaw - adjuvant PCV (after 54Gy) significantly increases OS. Median OS 13years. On sub analysis benefit only in IDH mut (both deleted and co-deleted) - Greatest gain = codeleted.
4) Fisher - low grade IDHmut AND 3 SATAN risk factors, 54Gy w/TMZconcurrent + adj appears better than controls.
5) Catnon - no-codeleted gliomas (anaplastic astrocytoma) has been assessed in the CATNON trial - final results pending, but so far OS benefit to 59.4/33 and adjuvant (but not current tmz).
6) Indigo trial - Grd II - Vorasidinib - IDH mut inhibitor - buys 2 years progression free.

Therefore:
For lower risk (SATAN <2) oligo it may be reasonable to delay adjuvant treatment, potentially give chemo 1st (Non-belivers,Baumert Trial).

Grd II - consider delaying RT with Vorasidinib (Indigo)
Grd III
- Oligo (co-deleted): 54/30 followed by PCV (Shaw trial)
- Astro: 59.4/33 with adjuvant Tmz. (CATNON)
(Grd 4 Astro - Stupp, as these used to be GBMs)

186
Q

Give the SATAN criteria.

Give your doses and treatment approach for low grade gliomas:

A

SATAN criteria: Size>6cm, Age>40, Tumour crosses midline, astrocytoma, Neurology. 3-5 Factors considered high risk.

For lower risk (SATAN <2) oligo it may be reasonable to delay adjuvant treatment, potentially give chemo 1st (Non-belivers, Baumert Trial).

Grd II - consider delaying RT with Vorasidinib (Indigo)

Grd III
- Oligo (co-deleted): 54/30 followed by PCV (Shaw trial)

  • Astro: 59.4/33 with adjuvant Tmz. (CATNON

(Grd 4 Astro - Stupp, as these used to be GBMs)

Brainstem/gentle dose - 50.4/28

PORT:
CTV= residual GTV + operative cavity based on post-operative imaging.

Inoperable:
GTV on fused T1!!!

187
Q

Dose and volumes for a grd III brainstem astrocytoma

A

50.4/28.!!!!

GTV = fused T1 MRI
CTV=GTV+1.5cm (trimmed to boundaries)
PTV=CTV+3mm

188
Q

Give treatment volumes for:
Low grade gliomas:

GBM

A

Glioma:
Inoperable: GTV on fused T1. CTV= GTV+1.5cm.

PORT: Residual GTV + operative cavity based on post-operative imaging. Use contrast to better deliniate tumour bed (obviously tumour wont enhance).

GBM:
GTV Bx only: the entire tumour on T1+C.
GTV post resection: T1+C residual GTV (including the MASS/tumour component on T2/FLAIR)+ tumour bed

CTV=GTV + 1.5cm trimmed to boundaries. Checked against T2 FLAIR abnormality which may be included beyond this expansion.

189
Q

Compare and contrast the radiology findings for:
Oligo, astro, GBM

A

CT:
1) Califications: Astro (20%), Oligo (80-90%!), GBM (rare)
2) GBM: hypodense centre if large due to necrosis, thickened (dense) irregular edges due to hypercellularity, oedema, marked mass effect.

MRI:
T1 - most tumours are hypointense on T1 (contour on this)

T2 (high water content): Most tumours are iso/hyper except the hypercellular (hypointense) = GBM and oligo.
GBM hypointense.
Oligo vs Astro - OLigo hypo, astro iso/hyper!!!!!

Contrast enhancing: Extra-axial tumours, GBM and PILOCYTIC ASTRO!!

190
Q

A 3 year-old girl presents with a biopsy confirmed primary neuroblastoma of the right adrenal gland. After the completion of all relevant staging investigations, she is found to have a poorly differentiated, MYCN amplified, INSS Stage IV disease

Expected OS with this treatment?

Dose and technique:

A

Consolidative radiotherapy to a total dose of 36Gy/20:
pre-op GTV 21.6Gy/12#, boost surgical bed to total overall treatment dose of 36Gy/#, 1.8Gy/#, 5/#s/wk to reduce LRR
Remember 1.5cm on pre-op and post op CTVs.

Pre-SIM
- Consider GA
Position: supine, arms up, head towards gantry- Immobilisation: vac bag
- Planning CT: 2mm slice CT from thoracic inlet to femoral head (overing entire lung, kidney for OAR DVH) Fusion: pre-op and post-op MRI
Target volume
- GTV(pre-op) = post-chemo and pre-op right adrenal tumour
- GTV(boost) = post-op gross residual disease
-
CTV21.6 = GTV(pre-op) + 1.5cm, clipped at anatomical boundaries

  • CTV14.4 = GTV(boost) + 1.5cm, clipped at anatomical boundaries
  • PTV=CTV+5mm

5yr OS 5-10%!!!

191
Q

A 4 yro has a left nephrectomy + node dissection for a favourable histology Wilm’s tumour. Pathology shows R1 resection but involvement of some left para-aortic nodes. There was no spill, or peritoneal involvement.

What are the indication for radiation therapy for a child with Wilm’s tumour? (2m)

A

Adjuvant RT to flank (to reduce risk of local recurrence):
Stage 3 (includes all the surgical stuff, Eg biopsy only, rupture, partial resection)
Unfavourable histo.
R1.

192
Q

Roa NOA Nordic Roa

Explain

A

Key RCT trials for older people w/GBM

Roa = phase III RCT - 40/15 = 60/30

NOA & NORDIC = TMZ = RT if methylated

Roa = Phase III - 25/5 = 40/15
IF
- age >64
Or
- age >50 and poor performance status (KPS 50-70)

193
Q

Evidence and dose for re-irradiation in:

GBM

LG-Glioma

A

GBM 35Gy/10#: Particularly in pts who have relapsed after a reasonable interval of local control (e.g>12mnths).

1 RCT phase II trial randomised pts to bevacizumab with or without reirradiation giving 35/10. No OS benefit but significant PFS benefit. UK RCR Grade A - 35/10

LG: 45/25.
UK RCR

194
Q

For cutaneous SCC PORT when would you consider treating nodes

A

2 or more node mets
Parotid node (unless parotidectomy and full nodal dissection)
Size: any lymph node >3 cm.
ENE
+Ve nodal margin or spillage
Close margins <2 mm to facial nerve.
Recurrent nodal disease.

195
Q

Right forehead 2.0cm x 1cm x 7mm thick poorly differentiated SCC. Peripheral margins are well clear, the deep margin is 0.3mm. There is evidence of perineural spread involving several nerves up to 0.2mm in diameter at the deep margin. There are no signs of further disease on examination or imaging. A decision is made to proceed with adjuvant radiation therapy.

Describe a suitable radiation therapy tdose fractionation schedule.

A

This is very high risk disease with extensive PNI with multiple nerves >0.1mm involved, deep invasion (>=6mm), positive deep margin (0.3mm, unexcised <2mm treat as +ve). PNI spread beyond foramina is incurable.

Therefore in 30#s:
R1(surgical bed) should get 63Gy
CTV63=bed + 1cm trimmed to boundaries.
CTV60 = HR PNI disease = CTV63+1cm (i.e 2cm expansion for PNI)
CTV56 (intermediate risk) = CTV60 + skin and subcutaneous tissues extending from inferior border of CTV60 to the supraorbital notch (course of V1 branches), and to the superior edge of preauricular nodes.
CTV54n (nodes and extended to stylomastoid foramen): Preauricular nodes with inferior portion extended to stylomastoid foramen.
CTV54elective - Discuss with patient: covering supraorbital course of nerve to potentially to fissure.

196
Q

What are considered adequate surgical margins for:
BCC
SCC

A

BCC 3-4mm

SCC and at least 4 mm for a low-risk SCC.

There are no published guidelines for a high-risk SCC. 6-10mm is accepted range.

197
Q

What are the controversies around the utility of adjuvant radiotherapy for thymoma?

A

There is overall a lack of trial data (no RCT) regarding radiation for thymoma. For at least stg >=II (Mazaoka stage) prospective database review suggests adjuvant radiotherapy is associated with improved OS. This extends to patients who have had an R0 resection - opening the question of whether adj RT is reasonable for most patients.

Lower grade (less than B2) have excellent 20year OS >=90%, and the role of radiotherapy in this setting is controversial given the outcomes without it, and the toxicities of RT especially late (cardiac, and second malignancy) given 20yr survival data.

198
Q

ii. What are the indications for radiation therapy in thymoma?

Doses and volumes

A
  • Inoperable - as local control or palliative.
  • Neoadj - to downstage tumour prior to resection.
  • Adjuvant: R1 and R2 resection, consider in R0. Thymic carcinoma.
  • Palliative: radioresponsive, decrease mass effect.

NeoAdj = 45/25
R0 thymic carcinoma = 45/25 to thymic space

R1= 54/27 - bed and thymic space

R2 = 54/27 thymic space and bed + 60Gy boost (to GTV + 5mm)

199
Q

Discuss the role of systemic therapy in the management of metastatic prostate cancer. Include in your answer the class of agents used, duration of therapy and justification for use. 5 marks

A
  • Anti-androgen medications - Typically non steroidal anti-androgens (e.g. Bicalutamide): In hormone naive pts: part of initial therapy + GnRH agonist +/- abiraterone (STAMPEDE ADT+enzalutamide + abiraterone no benefit). Alternatively can be used for 3-4 weeks as part of commencement of GnRH agonist - to prevent testosterone spike from initial LH surge. Can also be used when progression despite GnRH agonist (“total androgen blockade) to prolong PFS. ADT+ Apalutamide superior DMFS (SPARTAN). Used life-long or until significant progression.
  • GnRH agonist (e.g Goserelin 10.8mg s/c q3monthly), provides a PFS, OS and symptomatic benefit in patients with newly Dx metastatic prostate cancer, and is part of multimodal therapy as above. Duration life-long, or until progression, or in the setting of stable disease intermittent (restart when >10ng/ml).

-Androgen synthesis inhibitor - Abiraterone: Improves OS and PFS (STOPCAT meta-analysis)- used in the following contexts - as initial therapy for newly Dx metastatic disease (in combination with ADT but not ADT+Enzalutimide STAMPEDE), or when progression on ADT/disease progression.

-Chemo - Typically Taxane chemotherapy - Docetaxel, improves PFS and OS (CHAARTED phase III RCT OS increase from 48 to 58months) when part of initial Mx of metastatic disease. Also increases OS for N+ and locally advanced (STAMPEDE). 75mg/m.sqr (65mg for Asians) until response (up to 10 cycles).

200
Q

For rectal Ca, when may organ preservation TNT be attempted? What is the outcome?

A

OPERA Phase 2.- Stg II-III up to T3bN1 (3 or less regional nodes), 4nodes=N2.
They maxed out at 5cm T size.

FOLFOX/CAPOX -> Chemo RT
may not be as good as
ChemoRT -> FOlfox Capox

3yr organ preservation 80%

201
Q

In general, with patients being treated for rectal cancer with external beam radiation therapy, how would you manage: (3m)

i. A hip replacement

ii. Excess small bowel dose within the field in a patient requiring neoAdj RT.

A

i. A hip replacement
- Avoid high energy beam (e.g. 18MV)
- Avoid beam entry through hip prosthesis
- Contour artefact from hip prosthesis on planning CT and adjust densities in RT planning dose calculation
- Get physicist involved
- Review proximity between tumour to prosthesis
- Oblique kv imaging for treatment verification

ii. Excess small bowel dose within the field in a patient requiring neoadjuvant radiation therapy.
- Check persistent over more than 1 fraction.
- Re-SIM with full bladder to push bowel further out of field
- Most conformal technique (e.g. IMRT/VMAT) and prioritize dose constraints to bowel
- Prone position
- Trim superior PTV expansion, and consider compromise coverage more superiorly (if safe to do so)
- Daily CBCT to monitor, re-plan if required
- Monitor GI toxicity, pre-med ondansetron

202
Q

Options for T3a-b N0 Rectum Ca

A

“Good” prog on MRI pts with minimal extension into perirectal fat (<15) may be considered for surgery only.

Short course
Long course
TNT.

203
Q

Doses and volumes for all brain mets (resected/cavity and un resecected).

Key planning goal for modern WBRT
Criteria for SRS/SRT:

A

WBRT 30Gy/10# with hippocampal sparing. Aim Hippocampus D100 <10Gy.

If 4 or less tumours and size <4.5cm

1) Cavity:
SRS 12-20Gy
<30cc 27Gy/3
>30cc 25/5 (note this is only dose different from defitive where 30/5)
HR CTV = residual, bed, tract.
2) Intact
SRS 16Gy-20Gy
<30cc 27/3 (single 27/1 is in use)
>30cc to less than <4.5cm
30/5.

Volumes CTV=GTV, PTV=CTV+2mm

204
Q

What is the go-to Watch and wait study for follicular lymphoma? What stages/grd? What outcomes?

Compare that with other data?

A

Standford study

2/3 patients not need Tx at 7 years. 85% 10yr OS.

Compare that with:
Taking all grades <3
RT 24Gy in 12 fraction + CVP/ R-CVP
- TROG 99.03 showed improved PFS with addition of CVP/ R-CVP
From 50% 5yr RFS to 60%.

I.e Stanford study shows RT alone gives you a 50% chance of relapse free survival.

205
Q

Dose and technique for adj Brachytherapy component of definitive Cervix:

A

24gy/3#s, 3days apart, 2#s/week. Timed with final 2 weeks of RT (aim OTT <49 days), no chemo on brachy days. HRCTV D90 to receive 100% of prescribed dose.

Pre-treatment
- Anaesthetic review
Applicator insertion
- Under GA
- Lithotomy position
- Examination under anaesthesia (EUA): to assessment of tumour response (from external beam radiotherapy),
cervix and fornix anatomy (for selection of appropriate applicator/ ovoid size)
- Prep and drape
- 18F 3-way IDC catheter inserted, 7-10ml balloon, bladder filled to 300-400mL normal saline
- Uterus sounded to determine length of tandem applicator
- Cervical os dilated
- Insert applicator (tandem and 2x ovoid) under ultrasound guidance
- Vaginal packing with gauze to hold applicator in situ
- Position checked with ultrasound
Planning Imaging: MRI pelvis performed with applicator in situ Target volume
- GTV = macroscopic tumour (based on EUA at applicator insertion and MRI)
- HR-CTV = GTV + whole cervix
- IR-CTV = HR-CTV + 1cm expansion Plan review
- Aim HR-CTV D90 of EQD2 85-90Gy with / of 10 (with assuming the entire volume received 45Gy in 25
fraction on EBRT)

  • Aim IR-CTV EQD2 65-70Gy
  • Review OAR dosimetry
    o Bladder D2cc<90Gy
    o Rectum/ sigmoid/ bowel D2cc<75Gy Treatment delivery
  • Treatment delivery in bunker, with patient in supine position
  • Treat with Iridium-192 radiation source
  • Applicator removed after each brachytherapy fraction
206
Q

Doses for unresectable and borderline resectable pancreatic cancer. Give key studies and benefit for both:

A

Unresectable Proper (note that the role of radiation is controversional, especially with modern chemo FOLFIRNOX nab-Paclitaxel):
50.4/28 - LAP07 improves LC no OS benefit
Consider MASTERPLAN Trial

Borderline: Approximately 1/3 can be made resectable.
45/25 or 36/15.

207
Q

Give the VMAT broad technique (not volumes OARS ect) for an intermediate medulloblastoma.

Also key parts of plan evaluation.

Through treatment what do you have to monitor?

A

Cranio-spinal irradiation to a dose of 23.4Gy in 13 fraction
followed by tumour bed boost to a dose of 30.6Gy/17# (to total dose of 54Gy/30#) (1.8Gy/#, 5#/ week). (NB HR is 36/20 see then boot to a total of 54Gy - i.e 18gy).

RT Technique: VMAT technique with 6MV photon
* 3 isocentres (cranial, upper spinal, lower spinal), junction at 4cm depth, no feathering

Plan evaluation:
* Ensure CTV D98>98%, and PTV D98>95%,
* Minimise hotspot: D2<105% of prescribed dose
* Homogenous 16Gy covering entire vertebra body

Additional
* Weekly FBE monitoring (given entire vertebral marrow irradiation)

208
Q

Endometrial Cancer concurrent chemo:

Adj chemo:

Evidence:

A

50mg/m.sq D1 week 1 and D1 week (i.e Not weekly!!!!!!!!)

Adj is Carbo Pac.

PORTEC 3:
OS and PFS benefit -
Especially stage III, serous and p53abn

209
Q

Give Dose and approach to inflammatory breast cancer in each curative setting

A

Neo-Adj = inoperable after chemo.
MDACC 66/44, 1.5Gy/# BD. Breast and nodes 50.1Gy, GTV+1cm to 66Gy.

Adjuvant Residual/R1 resection:
MDACC 66/44, 1.5Gy/# BD. Breast and nodes 50.1Gy, GTV+1cm (or area of +ve margin) to 66Gy.

Full mastectomy and otherwise all good:
50/25 to everything.

Use Bolus!!

210
Q

Outline the relationship between dose and cardiac toxicity risk

A

The risk of cardiac event increases linearly 7.4% per Gy, with no threshold

This will likely be higher with the use of anthracycline-based chemotherapy and Herceptin

211
Q

Respiratory Motion Management Techniques Pros & Cons

A

1) Expiratory breathold: most reproducible, least movement. Cons: dificult, loose the benefit of reduced dose to oars.
2) DIBH: Often protects critical normal tissues. Minimal additional equipment. Con: limited by patient compliance/participation/lung function
3) Active Breathing control: Highly regulated breathing if tolerated. Con: may be poorly tolerated.
4) Abdominal drape/Compression: -less dependent on pt participation. can support smaller margins (lower ITV). Con - uncomfortable, may effect dosimetry for caudal tumours.
5) Vacuum drape: Less dependent on pt. Con: uncomfortable,long set up.
6) Real-time target tracking (e.g. MR linac): Great, super expensive.
7) Respiratory gating: Objective way to ensure reproducible position, increased treatment time, relies on tracking equipment.

212
Q

Recent evidence suggests which subgroups of locally advanced patients benefit from XRT post mastectomy and chemo?

A

More recent analysis in B-27 suggests 10-year risk of LRR is significant after neoAdjCTx (>10%) for the following patients:

tumor size >5 cm,
positive axillary lymph nodes,
younger age, and
incomplete response to NACT
portend a higher risk of LRR.

213
Q

For each of these systemic therapies give the side effects for which they are famous:
Taxanes
Cisplatin
Carboplatin
5-FU
Capecitabine
Etoposide
Ritux
Cetuxumab

A

Taxanes: Perif neuropathy - cardiomyopathy if used with Acanthracycline.

Cisplatin: Neuropthy, ottotoxicity, nephropathy, N&V

Carboplatin: Myleosuppression

5-FU: Mucocitis, hand foot and mouth - like rash, coronary artery spasm
Capecitabine: Is converted to 5-FU

Etoposide: Myleosuppression

Ritux: Super COVID

Cetuxumab: Skin rash - 95% of the time.

214
Q

Most radiosensitive soft tissue sarcoma and dose

A

Myxoid liposarcoma.
Only needs 36/20.

215
Q

Give doses for complete concurrent and adj Temzolomide course:

A

Concurrent: Daily 75mg/M.sq

One month post:
Monthly cycles (conventionally 6 but more can be considered):
Cycle 1:
Day 1-5 = 150mg/M.Sq/day
Cycle 2 onwards:
If tolerated increase to 200mg.

216
Q

Describe a whole brain technique for multiple brain mets:

A

Whole brain RT 30Gy/10#s, 3Gy/# VMAT technique with hippocampal sparing. 6MV photons. Prescribed to D50. Would include down to C2 if post fossa mets.

Pre sim: dex, analgesia, anti seizure as required.

Sim: Supine, head towards gantry, knee and ankle supports. Immob with thermoplastic mask.
1mm Slice CT scalp vertex to C7. Planing MRI (T1) to help deliniate hippos.

Volumes:
CTV = whole brain.
Hippocampal Avoidance Zone = bilateral hippos +5mm.
PTV = CTV+5mm with the hippocampal avoidance zone subtracted from this volume.

Goals:
1) Hippo:
Aim max dose <16Gy
Min dose <9Gy.
Blah.

217
Q

For a pre-menopausal lady, what should you consider before offering WBRT +/- hippo sparing?

A
  • Prego, or up the duff.
  • For small volume asymptomatic disease, consider central nervous system (CNS) penetrating systemic therapy, if available.
  • Previous radiation therapy to the same site.
  • Cardiac implantable electronic devices (CIEDs).
  • Concurrent cytotoxic therapy.
  • Non-rheumatoid collagen vascular disorders.
  • Degenerative neurological conditions e.g. MS
  • Possible hydrocephalus

Patients with favourable prognosis (>4 months life expectancy). Consider Brain metastases palliative EBRT hippocampal avoidance whole brain to decrease the risk of neurocognitive decline.

218
Q

For defitive skin cancer treatment - what is the lowest fraction (least visits) dose schedule?

A

35Gy/5#, 7Gy/#, 5#s/week. (EQD2 52Gy)

219
Q

Field margins for cutaneous SCC:

With key modifications

SXR PTV?

A

“Guidelines vary”

1cm radial (unless PD the 1.5cm), 0.5mm below estimated depth. If T3 or higher deep margin is 1cm.

If PNI then 2cm expansion.

This is trick question - The above margin should include sub clinical extension and margin for setup errors and penumbrum (which is 0.5mm for SXR)

220
Q

An example of an SXR setup to achieve a 5mm D90 for 2cm lesion.

A

Prescribed to surface.
Select SSD 20cm, Beam energy 120KV, AL filter 5mm

221
Q

How would you verbally give a skin electron plan.

A

“I would prescribe to the 90% isodose.”

“6MEV and with 1cm bolus, I would expect 90% Isodose at 1cm”

“9MEV and with 1cm bolus, I would expect 90% Isodose at 2cm”

222
Q

Prognostic factors for BCC:

Efficacy of topical treatment (and dose of preferred topical)

A

Patient: Immunosupression, Gorlin syndrome
Tumour: >5cm (50% 5yr LCR), location in “H zone”, Morphoiec, infiltrative, basosquaromus, recurrent, PNI.

Treatment: R1 30% 2yr recurrence.

223
Q

The QUARTZ trial:

A

Phase III NSCLC: Best supportive care vs WBRT in patients unsuited to resction or SRT/SRS.

No difference in OS, QOL, dex dose.
Subanalysis suggestive of a survival benefit age <60.

224
Q

For each situation for which WBRT (+/- hippo spare) may be considered comment on the evidence/benefit:

A

WBRT alone (where not appropriate for SRS/T):
Main goal is to palliate Sx of brain mets/oedema, and prevent deterioration of neurologic function. Despite much investigation/meta-analysis, little evidence for OS benefit.
The QUARTZ trial (unresectable/not for SRS NSCLC): no better than best supportive care except Pts <60.

Adj: Increased LC, No OS bene, worse cog and QOL. No benefit whatsoever for melanoma (Hong).

SRS+WBRT -

225
Q

48F Left 28mm, poorly differentiated SCC of anterior tongue, x2 LEFT level 2 nodes. Lingual nerve invasion.

Hemi-glossectomy:
1mm margin, left neck dissection nodes fully excised.

What do you do?

A

(Note about +ve margins. NCCN: <1mm is positive, <5mm = close).

Given lesion PD and node +ve I would consider 1mm +ve. Can consider concurrent Chemo.

Therefore I would treat EBRT, VMAT SIB technique to a maximum dose of 63/30, 5#s/week. Prescribed to D50.

Pre sim:
Dental review and necessary work
Consider PEG
Analgesia/PRN/anxiolytics as needed.
Dietician
Smoking cessation
and other allied health such as psychology as needed.

Sim:
Supine, head towards gantry, in neural position, arms by side shoulders lowered. Immobilisation with thermoplastic mask, knee and ankle supports.
2mm slice CT+contrast, vertex to upper thorax. Fused w/planning MRI and pre-operative imaging.

Volumes:
CTV63= Tumour bed based on imaging expanded by 5mm and trimed to boundaries.
CTV60=CTV63+1cm trimmed to boundaries.
CTV56=dissected nodal levels AND lingual nerve to the stylomastoid foramen - could consider path of V3 to foramen.
CTV54 = CTV56 + undisected bilateral nodes: 1a, 1B, II, III and IV. I would treat level V on ipsilateral side and omit IIa on the contraleral side.
PTV = CTV54+7mm

Daily CBCT matched to bone with soft tissue review.
OARS (in EQD2): Brain Dmax<60Gy, lens Dmax<6Gy, retina mean <45Gy, Lacrimal duct Dmean<30Gy, Parotid mean<26Gy, mandible avoid hotspots, cervical eso mean<30Gy,

226
Q

Indications for splenic radiation.

Dose schedule:

A

Indications = symptomatic splenomegaly:
- Malignant e.g. AML, NHL
- Cirrhosis related
- Extramedullary haematopoesis (e.g. due to myelodysplasia).
- Many more.

0.5Gy/# every second day with bloods every 2nd fraction. Titrated to clinical effect, total dose 10Gy.

227
Q

Gastric and gastroesophageal adenocarcinoma adjuvant EBRT chemoradiation:
Indications
Dose,
Trial,
Volumes

A

Indications: Node+ OR, T3 with R1, OR, T4.
Int trial - OS benefit, but role of RT debatable (CRITICS trial, success of FLOT).

45Gy/25#, 1.8Gy/#, 5#s/week, concurrent with capecitabine and cisplatin. Consider R1 boost to 50.4/33. VMAT technique

Volumes:
CTVboost = Tumour bed +1cm
CTV = CTVboost + Using table from TROG trial include CTV structures based on tumour location (gastroeso, proximal 1/3, body, distal 1/3). As per that guideline I would include pergastric and coeliac nodes and other nodal groups based on that table for each tumour location.
PTV = CTV+1cm.

228
Q

For whole breast RT give OARs. Not in EQD2, But:

1) 40/15
2) 26/5

A

2.67Gy#:
Lung V15%<15Gy
Heart V20 Gy <5%
Mean Heart Dose (MHD): <4 Gy

5.2Gy#:
Lung V15%<8Gy
Heart V7Gy<5%

229
Q

For the vulva:
What is included in the primary CTV (at minimum)

A

GTV+1cm
Whole vulva
Sub cutaneous mons bridge.

230
Q

OARS and constraints:

40/15 Boob

A

Lung V16Gy<15%
Contralateral lung V5Gy<10%
Heart V20Gy<5%
MHD<2Gy or 4Gy
Contra boob mean dose <1.5Gy

231
Q

OARS and constraints:

60/20 Prostate

A

Rectum V40Gy<40%
Bladder V40Gy<50%
Femur V40Gy<50%

Small bowel Dmax<43Gy
Large Dmax<44Gy

232
Q

Describe Lutetium-177 treatment:

A

Lutetium-177–PSMA-617:
- Beta-emitter with a half life of 6.7 days
- Appropriate patients = castrate resistant, met prostate ca, with PSMA avid disease (duh) and have already had at least one line of anti-androgen therapy AND one line of taxane chemo.
- Improves OS by ~4 months compared to “standard of care”
- no phase 3 RCT vs things like carbazitaxel or Radium-223 so the NICE group wasn’t impressed enough to fund it in the UK.
- Given by IV infusion, 200mCi once every 6 weeks for 4-6 cycles. Not necessary to do a post treatment uptake scan in most cases.
- Pretty well tolerated. SEs include dry mouth, nausea, fatigue.
- Risk of acute grade 3 myelosuppression modest (7-10%).

233
Q

Up to what stage may surgery be considered for cervix cancer?

Give the Peters criteria:

Give the Sedlis Criteria:

A

Disease localised to the cervix AND less than 2cm (Stage IA and IB1) OR IIA (upper 2/3rds of vag)
NB:
1B2 (limited to the cervix but big = 2 to 4cm) = Chemo RT

Peters (for post op chemo) = 3Ps = Parametrial Invasion, Pos Margin, Pos Node

Sedlis (for adj RT) = SDLvi = based on size, depth of stromal invasion, LVI

234
Q

Give the key decision variables for RT dose/approach for:

1) Rhabdo
2) Wilms
3) Ewings

A

Rhabdo (Surg->VAC-RT or not if SIOP):
Site (H&N/GU = 50.8Gy), Histo (RO alveolar 36/20), N+ (41/.4), Resection (alveolar or Embryonal R1 both get 36/20, R = 50.4).

Wilms (NWTS is upfront surg +/-RT +/-Chemo):
Stg 3 = adjRT = surgical events - Bx (seeding), seeding, piecemeal resection, STR, Rupture.
Fav Histo and Stg 3 = 10.8/6Gy
Anaplasia = Unfav Histo = 19.8Gy/11#, R1/R2 = Boost 10.8/6
WART (10.5/7) if Spill, Peritoneal Seed, Ascites Cyto+, Rupture (SPAR).
Whole Lung if Mets: 12/8 (do SIOP approach, if Cr no RT to lung)

Ewings (VCDA-IE):
Response to chemo (<90 or >90%)
Surgical Events: R1/R2Spill rupture, Margin <1cm
Unresctable - neoAdj 45/25
Poor (<90%) response to chemo = prechemo GTV to 54/30
Good = 45/25 to prechemo, boost postchemo to 54/30.